Vous êtes sur la page 1sur 63

FISICA I

GUIA DE TRABAJOS PRACTICOS


2014
MODULO II
CLASES 1-14

Cu rso d e Fs ica I

Se gu nd o Cu a t rime st re de 20 14

FISICA I
MODULO II
CLASE 1
Objetivos de la clase:
- Tratamiento del concepto momento de la cantidad de movimiento (llamado tambin
momento cintico o momento angular) sobre los modelos partcula y sistema de partculas.
- Introducir los conceptos de momento de una fuerza
- Extender el concepto de momento cintico a un sistema de partculas.
- Introducir la importancia de la distribucin de masas en la dinmica de las rotaciones sobre
los mencionados modelos.
Para poder trabajar en los temas de esta clase se requiere buen manejo de:
Vectores
Producto vectorial,
Leyes de Newton
Concepto de conservacin en el tiempo de una magnitud fsica
Ejercicio 1:
Objetivo: Aprender a calcular el momento angular.Revisar y fijar el carcter vectorial del
mismo.
Las figuras a y b esquematizan dos tramos planos y distintos de una carretera. Imaginemos un
automvil desplazndose con rapidez constante por dichos tramos. a) Para cada uno de los
casos, discuta si el mdulo del momento angular del automvil respecto del centro del crculo:
aumenta, disminuye o permanece constante, al pasar de un tramo recto al otro. b) determine la
direccin del momento angular para cada uno de los casos.

Fig. a

Fig. b

Ejercicio 2:
Objetivo: Aprender a calcular el momento angular. Emplear la relacin entre el momento de las
fuerzas actuantes sobre el sistema y la variacin del momento angular del sistema. Determinar
las condiciones necesarias para que se conserve el momento angular.
Un nio hace girar una pelota atada a un hilo sobre una mesa, en un momento el hilo se corta
saliendo tangencialmente a una velocidad de modulo 10 m/seg, si el hilo en el momento de
cortarse tena una longitud de 40 cm. Determine: la cantidad de movimiento, antes y despus de
cortarse el hilo, se conserva? Determine el momento angular de la pelota respecto de la
posicin inicial o centro de la circunferencia que describe; antes y despus de cortarse el hilo,
se conserva? Que sucede con la energa del sistema?

Cu rso d e Fs ica I

Se gu nd o Cu a t rime st re de 20 14

Ejercicio 3:
Objetivo: Estudiar el movimiento de un cuerpo bajo la accin de una fuerza central, a partir del
anlisis del momento de dicha fuerza y del momento angular del cuerpo.
La Tierra describe una rbita elptica alrededor del Sol, estando ste en uno de los focos de la
elipse. Cuando la Tierra est en la posicin ms alejada del
Sol (afelio) el 2 de julio, su distancia al sol es de 1,52 x10 11
m/s y su velocidad orbital es de 2,93 x 104 m/s.
Halle su velocidad orbital en la posicin ms cercana al Sol
(perihelio) aproximadamente 6 meses despus, cuando su
distancia al Sol es de 1,47.10 11 m. Justifique el
procedimiento.
Ayuda: analice la/s fuerza/s actuantes sobre la Tierra.
Cul es la magnitud del momento que ejerce/n respecto del centro del sol?
Datos: MTierra = 6 x 1024 kg.

MSol = 2 x 1030 kg.

Ejercicio 4:
Una parte de un sistema explorador enviado a un planeta de masa 1/3 de la de la Tierra e igual
radio dispara un proyectil con una velocidad v que forma un ngulo con la horizontal y
alcanza nuevamente la superficie del planeta a una cierta distancia del punto en el que se
efectu el disparo. Determine el momento angular del proyectil respecto de la posicin inicial:
a) en el momento en el que es disparado, b) en el momento en que alcanza su altura mxima, c)
en un momento inmediatamente anterior al instante en el que choca contra la superficie del
planeta, d) Si no hubiera hecho las cuentas necesarias para contestar los puntos anteriores,
tendra elementos para determinar si el momento angular se conserva?. Justifique sus
repuestas (Ayuda: Recuerde que para llegar a la definicin de momento angular se trabajo en
un marco de referencia inercial y que el valor del momento angular depende del punto respecto
del cual ste se determina)
Ejercicio 5:
Dos partculas se mueven en sentidos opuestos a lo largo de una lnea recta como se muestra en
la figura. La partcula de masa m se mueve hacia la derecha con velocidad v, mientras que la de
masa 3m se mueve hacia la izquierda con la misma rapidez. Cul es el momento angular total
del sistema formado por las dos partculas respecto del punto O, el punto A y el punto B?

Cu rso d e Fs ica I

Se gu nd o Cu a t rime st re de 20 14

Ejercicio 6:
El sistema de masas y cuerdas de la figura est apoyado sobre una mesa lisa y gira alrededor del
punto fijo O, con velocidad angular = constante. Las masas de las sogas son despreciables
frente a m y M. a) Hallar el momento angular del sistema respecto de O. b) Si en un dado
instante se corta la soga que une m con M, qu sucede con las cantidades de movimiento
y
? Y con el momento angular total? c) Posteriormente la masa M ingresa a una superficie con
rozamiento. Se conserva entonces el momento angular total del sistema? d) (Opcional) Para el
sistema de ambas masas en la situacin inicial, probar que se verifica que
.

Ejercicio 7:
Objetivo: Problema de integracin conceptual
Un bloque de masa M= 10 kg est unido a uno de los extremos de una varilla de masa
despreciable y longitud 2 m. El otro extremo de la varilla est pivotado a la superficie horizontal
sobre la que descansa el bloque. El pivote permite una rotacin que puede considerarse libre (es
decir sin rozamiento) alrededor de l. Un proyectil de masa m = 200 gr y velocidad v = 200 m/s,
paralela a la superficie y perpendicular a la varilla, se incrusta en el bloque. Si el roce del bloque
con la superficie puede despreciarse, a) qu tipo de movimiento efectuar el bloque despus de
la colisin? b) Determine las magnitudes necesarias para describirlo. c) Podra predecir la
posicin del bloque t segundos despus del momento de la colisin? d) Se conserva el
momento angular durante la colisin? e) Se conserva la cantidad de movimiento lineal durante
la colisin? f) Se conserva la energa mecnica durante la colisin? Justifique sus respuestas
aclarando cul es el sistema que esta analizando al responder cada pregunta.
Ejercicio 8:
Objetivo: Analizar la dinmica de la rotacin respecto de un eje fijo de un sistema de partculas
en el que la distancia entre partculas permanece fija. Ver la influencia de la distribucin de
masa respecto del eje de rotacin en la evolucin temporal del sistema.

m1

m2
L

El sistema de la figura consiste en dos esferas de volumen pequeo y densidad diferente, unidas
por una varilla de masa despreciable y longitud L. El conjunto descansa sobre un plano
horizontal liso.
Caso 1). El sistema se pivota en m 1 y sobre m2 se ejerce una fuerza de mdulo constante
y normal a la varilla.
Caso 2). El sistema se pivota en m 2 y sobre m1 se ejerce una fuerza normal a la varilla y
de igual mdulo que en el caso 1

Cu rso d e Fs ica I

Se gu nd o Cu a t rime st re de 20 14

El cociente entre la aceleracin angular que adquirir la varilla en el caso 1 y la que


adquirir en el caso 2 ser mayor, igual o menor que 1?. Justifique su respuesta
Ejercicio 9:
Una partcula de 0,5Kg est atada al extremo de un hilo y se mueve en
una trayectoria circular de 0,5m de radio sobre una superficie horizontal
sin friccin. El hilo pasa por un agujero en la superficie. Inicialmente su
otro extremo se mantiene fijo. Si se tira lentamente del hilo, de forma
que el radio disminuya a 0,35m, hallar
a) Cmo vara la velocidad angular , sabiendo que inicialmente w i = 2
s-1?. Indique claramente qu principios utiliz
b) El valor de la tensin sobre la cuerda antes y despus de cambiar el radio.
Bibliografa
-

R. Resnick and Halliday. Fsica. 3 y 4 edicin. Captulo 12 (incisos 1 a 4).


P. Tipler. Fsica. Tomo 1. 2 edicin. Captulo 13 (incisos 1 y 2).
P. Tipler. Fsica. Tomo 1. 3 edicin. Captulo 8 (incisos 2, 5 y 7).
M. Alonso y E.J. Finn. Fsica. Captulo 7 (incisos 13 y 14)
R. Resnick and Halliday. Fsica. 4 edicin.

Cu rso d e Fs ica I

Se gu nd o Cu a t rime st re de 20 14

FISICA I
MODULO II
CLASE 2
Objetivos de la prctica:
Introducir:
1.- El modelo cuerpo rgido (sistema de partculas con las distancias entre las
partculas fijas).
2.- La relacin entre velocidad lineal y angular y entre aceleracin lineal y angular
para distintas porciones de masa de un cuerpo rgido que rota.
3.- El concepto de momento de inercia. Teorema de Steiner.
4.- La dinmica del cuerpo rgido en movimiento de rotacin.
Ejercicio 1:
En la figura se observa un sistema constituido por dos esferas, una de masa m y otra de masa
2m, unidas por una varilla de masa despreciable, ubicadas a una distancia L una de otra.
Determinar el momento de inercia respecto de un eje de rotacin perpendicular a la varilla que
pase por el CM.

2M

Ejercicio 2:
En la figura se observa un sistema constituido por cuatro esferas de igual masa y radio unidas
por varillas de masa despreciable, de modo tal de formar un cuadrado rgido. Empleando la
definicin de momento de inercia encuentre con respecto a qu ejes de rotacin el sistema tiene
el mayor y el menor momento de inercia. Considere diferentes ejes que pasen al menos por una
de las esferas.
M

L
L

Ejercicio 3:
Objetivo: Estudio de la rotacin de rgidos simtricos respecto a un eje fijo (poleas con masa).
Martn utiliza el dispositivo de la figura para elevar, con aceleracin constante, bloques de masa
m desde el piso hasta una cierta altura. El plano que utiliza es rugoso.
En el transcurso de la maniobra se pregunta si la tensin de la cuerda es la misma en ambos
lados de la polea. Si se puede suponer que: a) la masa de la cuerda en este sistema puede
despreciarse, b) la polea es cilndrica, homognea y maciza,
c) La polea tiene masa M y radio R Qu le contestara? Cmo podra justificar su respuesta?
Martn desea, tambin, encontrar la aceleracin con que sube el bloque por el plano. Qu
ecuaciones debera plantear?

Cu rso d e Fs ica I

Se gu nd o Cu a t rime st re de 20 14

Por ltimo Martn decide que en lugar de tirar l de la cuerda le va a colgar un cuerpo cuyo peso
sea igual a la fuerza que l hizo.
El tiempo que el bloque tardar en llegar a la parte superior del plano,
Ser mayor, menor o igual que el que hubiese tardado si Martn hubiera continuado tirando de
la cuerda?
Explique su razonamiento. Plantee las ecuaciones correspondientes.

Ejercicio 4:
Dos cuerpos de 3 y 5 Kg estn unidos por una cuerda que pasa por
una polea en forma de disco de 2 kg de masa y 20 cm de radio.
Ambos deslizan sobre planos inclinados de 30 y 45. Los
coeficientes de rozamiento entre los cuerpos y los planos inclinados
son 0.3 y 0.1 respectivamente. Calcular:
a) la aceleracin del sistema,
b) las tensiones de la cuerda,
c) la velocidad que adquieren los bloques cuando se desplazan 5 m a lo largo de los planos
inclinados respectivos, partiendo del reposo, utilizando consideraciones energticas.
Ejercicio 5:
Objetivo: Estudio de sistemas de partculas con vnculos. Trabajo y energa en rgidos en
rotacin. Problema de integracin conceptual.
En la situacin mostrada en la figura, est el sistema en
equilibrio si los coeficientes de roce, esttico y dinmico, entre
m1 y el plano son 0,2 y 0,15, respectivamente? La polea es un
m1
cilindro homogneo de masa 4 kg, mientras que m1 = m2 = 10 kg.
b) Si en esta situacin se libera al sistema a partir del reposo
desde la posicin mostrada en la figura, cmo puede predecir la
posicin del bloque m2 para todo tiempo t posterior? c) Escriba la
funcin r2(t) y realice un grfico cualitativo de la misma. d)
m2
Indique qu suposiciones y/o aproximaciones realiz para llegar
a la respuesta. Disctalas con sus compaeros de grupo y
controle con el docente a cargo. e) Si en lugar de liberar al
sistema desde el reposo se le imprimiera al mismo una velocidad
inicial de 2 m/s, cmo se modificara su respuesta b)?
f) Obtenga, por consideraciones energticas, la velocidad que tendr m 2 al entrar en contacto
con el piso, comprela con la velocidad de m 1 para ese mismo instante. f) Podra determinar el
impulso que el piso realiza sobre m2? Qu informacin adicional necesitara?..

Cu rso d e Fs ica I

Se gu nd o Cu a t rime st re de 20 14

3m

Ejercicio 6:
Objetivo: Estudio de sistemas de partculas con vnculos. Trabajo y energa en rgidos en
rotacin. Problema de integracin conceptual.
El resorte de la figura, de constante k=1N/m, no est
extendido cuando se libera al bloque, permitindole caer. Si la
posicin inicial del bloque es de 10 cm respecto del suelo:
a) Utilizando conceptos energticos determine con qu velocidad
llegar el bloque al suelo.
b) Plantee las ecuaciones dinmicas que le permitiran obtener la
misma magnitud.
c) Ambos mtodos permiten obtener la misma informacin?
El momento de inercia de la polea es de 0,8 kgm2.
Ejercicio7:
Una varilla de longitud L y masa m cuelga y puede rotar libremente alrededor de un
pivote que pasa por uno de sus extremos. Si se la suelta cuando est en posicin horizontal, la
varilla va a girar alrededor del pivote.
a) Determine la aceleracin angular de la varilla en el instante inicial y cuando pasa por el punto
ms bajo de su trayectoria.
b) Cunto vale la fuerza que hace el pivote sobre la barra en la posicin inicial?
c) Suponga ahora que la varilla se encuentra inicialmente en posicin vertical, con el pivote en
su extremo inferior y se la apartara de dicha posicin. Al pasar por la posicin horizontal
cul de las magnitudes calculadas en a) y b) habr cambiado?
Para discutir en el grupo y verificar si se maneja el concepto centro de masa
Analice la validez de la aseveracin: Se puede tambin considerar el movimiento de la varilla
como una rotacin respecto del CM ms una traslacin del CM.
Ayuda: Piense qu relacin se puede establecer entre la aceleracin angular con que gira la
varilla respecto del pivote, la aceleracin con que gira respecto del CM, y la aceleracin del
CM.
Ejercicio 8:
En el sistema de la figura, el bloque de 500 g se mantiene en
reposo cuando el resorte de constante 200 N/m est en su
posicin de equilibrio. Se suelta el dispositivo, el bloque
desciende 10cm y se detiene por accin del resorte. Calcular por
consideraciones energticas:
a) La velocidad del bloque cuando descendi 5 cm b) La
velocidad angular de la polea en ese punto.
Ejercicio 9:
El balde de un aljibe, de masa 3 kg, cuelga sujeto a una cuerda que est
enrollada en un eje cilndrico apoyado sobre cojinetes sin friccin. La masa
del eje es de 4 kg y su radio r = 0.4 m. Si se suelta el balde desde el reposo,
hallar: a) La tensin de la cuerda. b) Las aceleraciones de los cuerpos. c)
Utilizando consideraciones energticas, la velocidad del balde cuando
desciende 3 m. Justificar. El momento de inercia del cilindro respecto de un
eje que pasa por su centro de masa es ICM = Mcr2.

Cu rso d e Fs ica I

Se gu nd o Cu a t rime st re de 20 14

Bibliografa
Resnick - Halliday - Krane: Fisica Vol.I 4a.o 5a ed. CECSA
P. Tipler. Fsica Vol 1, cuarta edicin, Revert, 2001
Tipler y Mosca Fsica Vol. 1. Revert, 2005.
Sears Zemansky,Young, novena Ed. Fsica Universitaria Vol. I
Seway y Jewett, Fsica Vol. 1, 2004. Thomson.

Cu rso d e Fs ica I

Se gu nd o Cu a t rime st re de 20 14

FISICA I
MODULO II
CLASE 3
Objetivos de la prctica:
Introducir:
1.- La dinmica de la roto-traslacin.
2.- Conceptos energticos relacionados con sistemas en rotacin.
3.- Condicin de rodadura sin deslizamiento.
4.- Aspectos dinmicos y energticos relacionados con la fuerza de roce en la rototraslacin.
5.- Eje instantneo de rotacin.
Ejercicio 1: Para pensar en la casa
Las ruedas del auto
Analizar el sentido de las fuerzas que las ruedas del auto hacen sobre el piso y sus reacciones si
el auto tiene traccin:
a) delantera
b) trasera

Ejercicio 2: Para hacer en la casa


Objetivo: Revisin de modelos.
Examinar si alguno de los modelos vistos hasta el presente permite describir y predecir los
movimientos que observar en las experiencias propuestas a continuacin.
a) Haga rodar en un plano inclinado (con un ngulo no muy grande), soltndolos desde
la misma altura, cuerpos de diferentes formas: esferas, cilindros macizos, cilindros
huecos, aros, etc. (sera ideal que tuvieran igual masa y distinto radio, igual radio y
distinta masa, etc. Podra asegurar que llegan todos al pie del plano con la misma
velocidad?
Explique recordando la ley de cada de los cuerpos de Galileo.
b) Consiga dos objetos cilndricos iguales (por ej. dos rollos de papel o dos envases de

desodorante en aerosol), y colquelos sobre alguna superficie que forme un ngulo pequeo con
la horizontal (como se muestra en las figuras a y b). Ponga por escrito el tipo de movimiento
que observa en cada caso. Si observa diferencias entre ambos comportamientos, explique por
escrito a qu las atribuye.
Para hacer en clase: Discuta las observaciones y explicaciones anteriores con su grupo de
trabajo, escribiendo la conclusin obtenida, y analcela con el docente a cargo.

Cu rso d e Fs ica I

Se gu nd o Cu a t rime st re de 20 14

10

Ejercicio 3:
Objetivo: Tratamiento dinmico de la rodadura sin deslizamiento
Un cilindro homogneo de masa m y radio R rueda sin deslizar sobre una superficie
horizontal, que empalma con una rampa ascendente inclinada un ngulo con respecto a la
horizontal. El cilindro alcanza la base de la misma y asciende por ella, tambin sin deslizar.
Haga un dibujo esquemtico de la superficie sobre la que se mueve el cilindro.
a) Haga un diagrama de las fuerzas que actan sobre el cilindro durante su
movimiento en el plano horizontal indicando qu agente las ejerce (antes de decidir
si una fuerza est actuando o no sobre el cuerpo analice si el movimiento del
cuerpo es consistente con la accin de cada fuerza que haya incluido en el
diagrama).
b) dem durante el ascenso.
c) En algn momento, el cilindro se detiene y comienza a rodar cuesta abajo. Haga el
diagrama de las fuerzas que actan sobre el cilindro durante el descenso.
d) Calcule la aceleracin lineal del CM en cada uno de los tramos analizados en base
a los datos suministrados.
e) Cmo interviene la rugosidad de la superficie en el movimiento del cilindro?
Cambiara el tipo de movimiento si la superficie de la pista fuese lisa?
f) Plantee el diagrama de las fuerzas que actan sobre el cilindro cuando ste vuelve a
encontrarse sobre la superficie horizontal.
g) Repita d) empleando el concepto eje instantneo de rotacin
Ejercicio 4:
Objetivo: Tratamiento energtico de la rodadura sin deslizamiento
El cilindro del problema anterior se detiene cuando alcanza una altura h respecto del
tramo horizontal.
a) Calcular qu velocidad tendr el CM cuando el cilindro regrese al pie del plano.
i) por consideraciones energticas, trabajando con el centro de masa.
ii)
lo mismo utilizando el concepto de eje instantneo.
iii) cinemticamente, utilizando la aceleracin calculada en el ejercicio 3.
b) Repetir a.i) y a.ii) para una esfera.
c) Por qu puede utilizarse el principio de conservacin de la energa mecnica cuando se
analiza una rodadura sin deslizamiento?
d) Se conservara la energa mecnica si hubiera deslizamiento? EXPLIQUE
Ejercicio 5:
Objetivo: Analizar el valor y direccin de la fuerza de roce en rodadura. Identificar la
influencia de la misma en el tipo de movimiento del sistema.
Dos cilindros macizos estn unidos por otro, tambin macizo, corto y de menor radio,
como se muestra en la figura. Cuando se enrolla una cuerda en este ltimo, y se tira de la misma
mantenindola paralela al piso, el sistema comienza a rodar.
a) Describa qu tipo de movimiento espera que realice el CM. Explique qu funcin
representara mejor la posicin del CM para tiempos posteriores. Haga un grfico cualitativo de
posicin en funcin de tiempo.
b) Qu sentido tendr la aceleracin angular?
c) Las respuestas a las preguntas anteriores dependen de la ubicacin de la cuerda? (Ver
figura)
d) Analice qu condiciones tienen que cumplirse para que no haya deslizamiento
e) Compare sus predicciones con las de sus compaeros de grupo
Cu rso d e Fs ica I

Se gu nd o Cu a t rime st re de 20 14

11

Solicite al docente a cargo un dispositivo similar al descrito, que se encuentra en el gabinete de


fsica, o algn otro dispositivo similar (yo yo, carrete de hilo; etc.) y verifique si sus
predicciones fueron correctas. Discuta los resultados en el grupo. Pregunte por un programa
de simulacin sobre situaciones similares.

Ejercicio 6:

Ro

h
Una esfera de masa M y radio R parte del reposo desde la posicin indicada en la figura, y rueda sin deslizar por una pista. sta,
consiste en un tramo circular y luego un tramo horizontal situado a una altura h por encima del piso.
a)Qu fuerzas actan sobre la esfera: a1) mientras recorre el primer tramo (circular)?, a2)
cuando se mueve en el tramo horizontal?, a3) despus de dejar la pista horizontal y antes de
llegar al suelo?
b) Cul es la energa mecnica cuando llega al tramo horizontal? c) A qu distancia
(horizontal) del fin de la pista choca la esfera con el piso?
c) Cul es la velocidad de la esfera en el instante inmediatamente anterior a su choque con el
piso.
c) Si esta situacin se diese en la superficie de Marte cambiara alguna de (todas) sus
respuestas?
Ejercicio 7:
Un cilindro uniforme de masa m 1=15kg y radio R=50cm rueda sin deslizar por un plano
horizontal sujeto a una cuerda que se encuentra unida en el otro extremo a un cuerpo m 2=3kg el
cual pende de una polea de masa despreciable. El sistema se libera del reposo .a) Cul es la
aceleracin de m2? b) Cul es la tensin de la cuerda? c) Cul es la aceleracin angular de m 1?
d) Aplicando conceptos energticos calcular la velocidad de m2 cuando la cuerda se desenroll
una longitud l=1m

Ejercicio 8:
El Rulo
Nuevamente nos ubicamos en una pista que tiene un trayecto en forma de rizo o rulo (loop).
Pero ahora, en lugar de tener una pelotita que podemos modelizar como partcula, tomamos el
modelo de cuerpo rgido. Si analizamos la velocidad de la pelotita en el punto ms bajo, es la
Cu rso d e Fs ica I

Se gu nd o Cu a t rime st re de 20 14

12

misma que cuando utilizbamos el modelo de partcula? Analizar por medio de consideraciones
energticas. El momento de inercia de la esfera es: Iesf = 2/5 m R 2

Ejercicio 9:
Un cilindro con una masa de 10 kg rueda sin deslizarse sobre una superficie horizontal. Si el
modulo de la velocidad de su centro de masa es 10 m/s, calcular: (a) la energa cintica de
traslacin de su centro de masa, (b) la energa de rotacin alrededor de su centro de masa y (c)
su energa total.
Ejercicio 10:
Un aro de 3.13 m de radio tiene una masa de 137 kg. Rueda sin deslizar a lo largo de un piso
horizontal de modo que su centro de masa tiene una velocidad 0.153 m/s. Cuanto trabajo debe
realizarse sobre el aro para detenerlo?
Ejercicio 11:
El sistema de la figura consta de una polea formada por dos discos coaxiales soldados de masas
550 y 300g radios R=8cm y R=6cm, respectivamente. Dos masas de m=600g y m=500g
cuelgan del borde de cada disco. Calcular:
a) La tensin de la cuerda que sostiene al cuerpo m.
b) La aceleracin de cada masa y de la polea
c) La velocidad de cada cuerpo cuando el cuerpo de masa m haya descendido 3m
partiendo del reposo

Cu rso d e Fs ica I

Se gu nd o Cu a t rime st re de 20 14

13

Bibliografa
Resnick - Halliday - Krane: Fisica Vol.I 4a.o 5a ed. CECSA
P. Tipler. Fsica Vol 1, cuarta edicin, Revert, 2001
Tipler y Mosca Fsica Vol. 1. Revert, 2005.
Sears Zemansky,Young, novena Ed. Fsica Universitaria Vol. I
Seway y Jewett, Fsica Vol. 1, 2004. Thomson.
Si tiene alguna duda sobre la bibliografa que est usando consulte con los docentes o solicite
algn libro del paol.

Cu rso d e Fs ica I

Se gu nd o Cu a t rime st re de 20 14

14

FISICA I
MODULO II
CLASE 4
Objetivo
Introducir:
1. Las Condiciones que deben cumplirse para que exista Equilibrio Rotacional
2. Las Condiciones que deben cumplirse para que exista Equilibrio Esttico
Cuestionario Gua:
Qu entiende por equilibrio? Utilice su definicin para establecer: Qu condiciones
deben cumplirse para que un cuerpo est en equilibrio rotacional? Puede analizar el
cumplimiento de la condicin de equilibrio rotacional respecto de un eje arbitrario que pase por
cualquier punto del cuerpo? Justifique esta ltima respuesta. Qu entiende por equilibrio
esttico? Qu condiciones deben cumplirse para que pueda decirse que un cuerpo rgido est
en equilibrio esttico? Defina el centro de gravedad. Qu entiende por una cupla? Cul es el
valor del torque ejercido por una cupla? Depende este torque del punto de aplicacin de cada
una de las fuerzas que componen la cupla?
Ejercicio 1:
Objetivo: Revisar el manejo de la dinmica de sistemas que se desplazan en una trayectoria
circular y la necesidad de emplear un marco de referencia inercial para poder analizar las
condiciones de equilibrio.
Un camin debe transportar una caja ms alta que ancha con un contenido frgil (cuyo
CM coincide con su centro geomtrico). El camino por donde transita el camin tiene muchas
curvas, aunque el conductor no teme que la caja deslice, porque la superficie es suficientemente
rugosa. En cambio, s teme que la caja vuelque. Por ello comienza a hacer hiptesis sobre esta
posibilidad, para calcular a qu velocidad mxima deber tomar cada curva. Dicha velocidad,
depender de:
a)
b)
c)
d)
e)
f)

la masa de la carga?
de su espesor?
de su altura?
de la relacin entre el espesor y la altura?
del coeficiente de roce?
del radio de la curva?

De qu manera depender de cada variable?


Haga un diagrama de fuerzas, plantee las ecuaciones necesarias y corrobore si las
hiptesis son correctas. (Ojo: cul va a ser su eleccin de sistema de referencia? Con relacin
a qu punto va a calcular los momentos de las fuerzas actuantes?).
Encuentra alguna relacin entre este ejercicio y la experiencia anterior?
Ejercicio 2:
Una barra uniforme de masa m y longitud l est sujeta a una pared mediante un cable en un
extremo y un pivote sin rozamiento en el otro. a) Calcular la fuerza que ejerce el pivote sobre la
barra. b) El cable se corta repentinamente, y la barra comienza a rotar alrededor del pivote (que
se supone sin rozamiento). En el instante en que la barra pasa por la posicin horizontal,
calcular b1) su aceleracin angular, b2) su velocidad angular, y b3) la fuerza ejercida por el

Cu rso d e Fs ica I

Se gu nd o Cu a t rime st re de 20 14

15

pivote. Ayuda: en el tem b2), utilizar la conservacin de la energa mecnica; en el b3), tener en
cuenta que el centro de masas de la barra se mueve alrededor del pivote con un movimiento
circular no uniforme.

Ejercicio 3:
Un audaz motociclista acelera su moto logrando despegar la rueda delantera del piso, cules
son las fuerzas que actan sobre el sistema moto motociclista en el instante en que la rueda
deja de estar en contacto con el piso?
a) Realice un diagrama y analice la condicin que debe cumplirse para que el sistema est en
equilibrio de rotacin pero no de traslacin.
b) Si la distancia entre las ruedas es L, el CM dista L/3 de la rueda delantera y est a una altura
h del suelo, calcule el mnimo coeficiente de roce necesario para que el sistema est en
equilibrio de rotacin. Es dicho coeficiente esttico o dinmico? Es necesario conocer la
masa del sistema?
Ejercicio 4:
En el sistema de la figura el bloque A esquematiza una heladera comercial de 70 cm de ancho y
1,5 m de altura que debe ser desplazada. Para tal fin se utiliza un
aparejo consistente en una polea y una soga (ambas de masa
despreciable) y una masa B de 50 kg. Si el coeficiente de roce
A
entre la heladera y el piso es 0,4 y la heladera tiene una masa de
100 kg cul es la mxima distancia del piso a la que puede
sujetarse la soga para que la heladera deslice sin volcar?
Suponga que el CM de la heladera coincide aproximadamente con
su centro geomtrico.
Ejercicio 5:
Una escalera de dos hojas, una de ellas de masa m y la otra de masa 2 m, est
apoyada en una superficie rugosa. Cada una de las hojas forma un ngulo con la horizontal.
a) Analice las fuerzas que actan sobre la escalera.
b) Analice las fuerzas que actan sobre cada hoja.
c) Establezca una relacin entre la fuerza de roce y el ngulo para el cual la escalera comienza a
deslizar.
d) Calcule la fuerza que hace cada una de las hojas sobre la otra.
e) Qu funcin piensa que cumple la varilla o cadena que suele unir las dos hojas a una cierta
altura?

Cu rso d e Fs ica I

Se gu nd o Cu a t rime st re de 20 14

16

Ejercicio 6:
Un portn de 1700 kg. y de dimensiones segn se muestra
en la figura, est sostenido en la pared por medio de las bisagras
A y B. Su centro de gravedad coincide con su centro geomtrico.
Calcule las componentes vertical y horizontal de la fuerza
que hace la pared sobre las bisagras.
NOTA: el soporte inferior establece un vnculo simple, esto
es, slo ejerce una fuerza horizontal.
Ejercicio 7:
El letrero de una posada pesa 1000 N, est colgado como
se muestra en la figura. El brazo que lo soporta, pivotado en la
pared, pesa 500 N., el sistema est mantenido por un cable que
no puede someterse a una tensin superior a 2000 N.

Omb

a) Haga un diagrama de las fuerzas sobre el brazo.


b) Calcule la distancia mnima AB.
c) Cul es, en estas condiciones, la fuerza que hace la pared sobre el soporte en A? (Nota:
Recuerde que las fuerzas son vectores)
Ejercicio 8:
En la figura se muestra una tabla uniforme de masa M y longitud
L, colgada del techo de un galpn mediante dos cuerdas. De uno de los
extremos cuelga una caja de masa m. Encontrar el valor mximo de m (en
funcin de M) para que la tabla se mantenga en equilibrio.

1/3 L

Ejercicio 9:
Una tabla homognea de 9m y de 400N se apoya simtricamente sobre dos soportes que estn
separados por una distancia de 4,8m. Un hombre que pesa 640N parte de A y se dirige hacia la
derecha.
Graficar las fuerzas ejercidas por los soportes en A y en B en funcin de la posicin del hombre.

Ejercicio 10:
Una escalera uniforme de longitud L y masa m1 se apoya contra una pared sin friccin. La
escalera forma un ngulo con la horizontal. (a) Calcular las fuerzas vertical y horizontal que
ejerce el suelo sobre la base de la escalera cuando un bombero de masa m2 est a una distancia
x (paralela al piso) de la base de la escalera. (b) Si la escalera est a punto de resbalar cuando el
bombero est a una distancia d de la base de la escalera, Cul es el coeficiente de roce esttico
entre la escalera y el suelo?

Cu rso d e Fs ica I

Se gu nd o Cu a t rime st re de 20 14

17

Bibliografa
Resnick - Halliday - Krane: Fisica Vol.I 4a.o 5a ed. CECSA
P. Tipler. Fsica Vol 1, cuarta edicin, Revert, 2001
Tipler y Mosca Fsica Vol. 1. Revert, 2005.
Sears Zemansky,Young, novena Ed. Fsica Universitaria Vol. I
Seway y Jewett, Fsica Vol. 1, 2004. Thomson.
Si tiene alguna duda sobre la bibliografa que est usando consulte con los docentes o solicite
algn libro del paol.

Cu rso d e Fs ica I

Se gu nd o Cu a t rime st re de 20 14

18

FISICA I
MODULO II
CLASE 5
Objetivos
1 Introducir el concepto de Momento Cintico de un Cuerpo Rgido en rototraslacin.
2 Trabajar con el Principio de Conservacin del Momento Cintico
ATENCION: No olvide que el momento cintico es un vector, tenga presente que
proviene de un producto vectorial entre dos vectores y que el producto vectorial no es
conmutativo.
Ejercicio 1:
En un incendio Ud. necesita cerrar rpidamente la puerta de una habitacin para aislarla
del humo y se encuentra imposibilitado de alcanzar el picaporte.
Dispone de una pelota de goma y de un trozo de igual masa de masilla.
a) Cul de los cuerpos le arrojara (con la misma velocidad) para conseguir su objetivo?.
(Suponga por simplicidad que la direccin de incidencia es normal a la puerta).
Sugerencia: Es conveniente tomar como sistema al cuerpo arrojado.
b) Tomando como sistema la puerta. En qu parte de la puerta conviene que impacte el objeto
lanzado? Explique su razonamiento.
Ejercicio 2:
La plataforma espacial ex sovitica Soyuz (sali antes de la disolucin de las
repblicas socialistas soviticas) viaja por el espacio exterior a velocidad constante (respecto
de qu?). En ella se encuentran dos astronautas, Yuri y Tatiana, ambos de 80 kg. Despus de
varios meses de aburridas observaciones cientficas, deciden acercarse entre s. Para ello ambos
se mueven con velocidades de magnitud 4 m/s respecto de la plataforma, siguiendo trayectorias
paralelas separadas 25 m.
a) Haga un esquema de la situacin
b) Calcule la cantidad de movimiento del sistema formado por los astronautas respecto de la
plataforma. Es esta magnitud constante en el tiempo? Justifique.
c) dem respecto del centro de masa del sistema formado por los astronautas. Qu
movimiento tiene el CM respecto de la plataforma?
d) Calcule el momento cintico respecto del CM. Se conserva? Justifique. (Ayuda: piense
qu condiciones tienen que cumplirse para la conservacin del momento cintico)
Cuando se encuentran uno frente al otro, Yuri lanza una cuerda (de masa despreciable) a
la que Tatiana se afirma fuertemente.
e) Describir el movimiento que realiza el sistema constituido por los dos astronautas. Qu
puede decir del comportamiento del momento cintico y de la cantidad de movimiento
respecto del tiempo? Explique su razonamiento.
f) Identifique las fuerzas a las que estn sometidos cada uno de los astronautas, y encuentre las
correspondientes aceleraciones.
Los astronautas intentan acercarse el uno al otro, para concretar algn tipo de encuentro.
g) Tendr lugar el encuentro?

Cu rso d e Fs ica I

Se gu nd o Cu a t rime st re de 20 14

19

h) Supongamos que la cuerda se corta cuando la distancia que los separa es de 10 m. (Por qu
se cortara?) Con qu velocidad y en qu direccin saldrn Yuri y Tatiana? Cmo ser
ahora el movimiento del CM?
i) Analice: qu sucede con la energa del sistema (defina claramente cul es) durante la odisea
de Yuri y Tatiana. Justifique.
Ejercicio 3:
Un hombre est de pie en el centro de una plataforma circular (sin friccin), manteniendo sus
brazos extendidos horizontalmente con una pesa en cada mano y girando alrededor de un eje
vertical con velocidad angular de 2 rev/s. El momento de inercia del sistema plataforma +
hombre respecto de este eje es de 10 kgm2. Cuando el hombre acerca las pesas hacia su cuerpo,
el momento de inercia disminuye a 4 kg m2 a) Cul es entonces la nueva velocidad angular de
la plataforma? b) Cul es la variacin de la energa mecnica experimentada por el sistema? c)
Cmo se explica fsicamente este cambio en la energa mecnica?
Ejercicio 4:
Una mujer de 60 kg est parada en el borde de una calesita (sin roce) de 1 m de radio que se
encuentra en reposo y cuyo momento de inercia respecto de su eje es I = 500 kgm2. La mujer
comienza a caminar por el borde de la calesita en el sentido de las agujas del reloj con una
rapidez constante de 1.5 m/s respecto del suelo. a) En qu direccin y con qu velocidad
angular se mover la calesita?
Ejercicio 5:
Una plataforma de densidad uniforme descansa sobre
una superficie sin roce. Dos caones iguales fijos a la
plataforma apuntan en direcciones opuestas disparando
simultneamente dos proyectiles de masa m, con velocidades
de mdulo v1 y v2=2.v1 respectivamente. El momento de
inercia de la plataforma y de los caones respecto de un eje
perpendicular a la plataforma y que pasa por el CM es I y la
separacin entre los caones L.
Qu tipo de movimiento efectuar la plataforma despus del disparo? Encuentre en
funcin de los datos disponibles las magnitudes cinemticas que describen el movimiento de la
plataforma despus del disparo. Justifique las ecuaciones utilizadas.
Ejercicio 6:
Una bala de 20 g que se mueve horizontalmente con velocidad v choca y queda incrustada en el
extremo inferior de una varilla de 20 cm de longitud y 0.5 kg. La varilla se encuentra
inicialmente en reposo en posicin vertical, suspendida por un pivote ubicado en su extremo
superior alrededor del cual puede girar libremente. a) Calcular la velocidad mnima de la bala
para que la varilla gire un ngulo de 180o
. b) Calcular la energa mecnica perdida en la colisin. c) Se conserva la cantidad de
movimiento del sistema bala + varilla en la colisin? En caso contrario, qu agente externo
ejerce una fuerza sobre el sistema? Qu direccin tiene esta fuerza? d) dem a), pero en el caso
de que la velocidad de la bala forma un ngulo de 30o con la horizontal.

Cu rso d e Fs ica I

Se gu nd o Cu a t rime st re de 20 14

20

Ejercicio 7:
Un disco (masa M= 4 kg y momento de inercia respecto de un eje
perpendicular al dibujo y que pasa por el CM I = MR 2) se encuentra
en reposo sobre una superficie horizontal de roce despreciable, cuando
es atravesado a una distancia d = .R de su CM, por un proyectil de
masa m = 0.2 kg y velocidad v = 40 m/s en la forma indicada en la
figura.
El proyectil atraviesa el disco, saliendo con una energa cintica
final igual a de la energa cintica inicial.
a) Prediga por escrito cul ser el movimiento posterior del disco y
escriba las ecuaciones que le permitan obtener las magnitudes que
describen dicho movimiento.
b) Compare con sus compaeros de grupo las predicciones y ecuaciones y acuerde con ellos
cules son las correctas y cmo se las justifica.
c) Discutan las conclusiones del grupo con el docente a cargo
Ejercicio 8:
Un disco de masa m1=1kg r1=0.3m que gira con =33rpm se acopla a otro de masa m2=350g y
r2=0.15m en reposo. Los ejes de rotacin de ambos discos coinciden.
a) Calcular la velocidad angular despus del acople para cada masa.
b) Determinar si la energa mecnica Em se conserva. Calcular esta ltima.
c) Si se supone que la froce dinmica acta en un radio medio de 1cm y que ambos discos
no deslizan, calcularla luego de recorrer rad considerando solo la variacin de energa
cintica

Ejercicio 9:
Dos esferas iguales de masas 6 kg y 20 cm de radio estn montadas como se indica en la figura,
y pueden deslizar a lo largo de una varilla delgada de 3 kg de masa y 2 m de longitud. El
conjunto gira libremente con una velocidad angular de 120 rpm respecto a un eje vertical que
pasa por el centro del sistema. Inicialmente los centros de las esferas se encuentran fijos a 0.5 m
del eje de giro. Se sueltan las esferas y las esferas deslizan por la barra hasta que salen por los
extremos. Calcular:
(a)La velocidad angular de rotacin cuando los centros de las esferas se encuentran en los
extremos de la varilla. (b) Hallar la energa cintica del sistema en los dos casos

Cu rso d e Fs ica I

Se gu nd o Cu a t rime st re de 20 14

21

3 /4 R

Bibliografa
Resnick - Halliday - Krane: Fisica Vol.I 4a.o 5a ed. CECSA
P. Tipler. Fsica Vol 1, cuarta edicin, Revert, 2001
Tipler y Mosca Fsica Vol. 1. Revert, 2005.
Sears Zemansky,Young, novena Ed. Fsica Universitaria Vol. I
Seway y Jewett, Fsica Vol. 1, 2004. Thomson.
Si tiene alguna duda sobre la bibliografa que est usando consulte con los docentes o solicite
algn libro del paol.

Cu rso d e Fs ica I

Se gu nd o Cu a t rime st re de 20 14

22

FISICA I
MODULO II
CLASE 6 (OPTATIVA)
Objetivo:
Estudio de sistemas que rotan alrededor de un eje mvil (con un punto fijo) desplazndose no
paralelo a s mismo
Ejercicio 1:
Todos o casi todos, hemos andado alguna vez en una bicicleta. Hemos observado que
para doblar necesitamos inclinar la bicicleta (tambin es el caso de las motos) hacia el mismo
lado de giro. Se han preguntado por qu? Tendr algo que ver con las magnitudes fsicas que
hemos visto hasta ahora?
a) Hacer un diagrama de fuerzas.
b) Comparar con el mecanismo para doblar de los coches.
c) Comparar con una moneda puesta a rodar. En qu condiciones la moneda dobla?.
Experimente.
d) Andando en bicicleta Sera posible doblar en una esquina si la superficie sobre la que se
est desplazando fuera totalmente lisa (por ej. si hay una mancha de aceite en el
pavimento)?
Ejercicio 2:
El ingeniero de mantenimiento de un avin quiere saber cul ser la fuerza a la que se
vern sometidos los rodamientos de la turbina cuando el avin utilice su mxima capacidad de
giro, doblando con un radio R = 100 m a una velocidad V = 180 km./h. El sistema de propulsin
consiste de una turbina que gira a 3000 r.p.m. y que pesa 160 kg., tiene un radio de giro de 30
cm y est montada sobre una bancada de 2m de largo, mientras la hlice pesa 15 kg. y su radio
de giro es de 65 cm.
a) Por qu cree el ingeniero que aparece dicha carga?
b) Que efectos adicionales al del desgaste normal tendr esta carga sobre el avin?
Actuar el mismo par si la turbina gira en sentido horario o antihorario? Qu precauciones
debe tomar el constructor?
c) Qu fuerza centrpeta ser necesaria para que el avin gire, considerando que su masa es de
3000 kg. Suponga que la fuerza de sustentacin es perpendicular a la superficie de las alas y que
depende del cuadrado de la velocidad del avin respecto del aire (esta es una primera
aproximacin bastante buena). Qu conclusiones (cualitativas) puede sacar de esto, en relacin
con la maniobra de giro?
Ejercicio 3: Para pensar
Qu relacin existe entre el momento angular, la necesidad de balancear las gomas de los autos
y los consejos del manual de uso de los lavarropas verticales que nos indican: trate de distribuir
la ropa alrededor del eje de rotacin?
Ejercicio 4.
Objetivo: Analizar el comportamiento de sistemas en los que los vectores momento angular y
velocidad angular no son paralelos

Cu rso d e Fs ica I

Se gu nd o Cu a t rime st re de 20 14

23

Un disco uniforme de 30 cm de radio, 3 cm de espesor y 5 kg rota con una velocidad


angular = 10 rad/s alrededor de un eje paralelo a su eje de simetra pero desplazado 0,5 cm del
mismo.
a) Qu fuerza neta aparecer sobre los rodamientos?.
b) En qu lugar del disco debe colocarse una masa de 100g para evitar el desgaste
c) Por qu no se puede balancearse una rueda en reposo?

Cu rso d e Fs ica I

Se gu nd o Cu a t rime st re de 20 14

24

FISICA I
MODULO II
CLASE 7
Objetivos de la clase:
- Introducir nociones de elasticidad esttica en slidos y fluidos
- Introducir el modelo de fluido ideal
- Estudiar fluidos ideales en equilibrio: Teorema general de la hidrosttica. Principio de
Arqumedes
- Introducir los efectos de superficie en el equilibrio de fluidos reales
Ejercicio 1:
Objetivo: Anlisis de cuerpos elsticos.
Un alambre de 135.00 m de largo y 4x10-5 m2 es estirado hasta alcanzar una longitud de 135.07
m. (a) Calcular la deformacin del alambre. (b) Si el alambre es de cobre, cul es el esfuerzo
necesario para producir esta deformacin? El mdulo de Young del cobre es de Y=10*10 10 N/m2
(c) Cul es la tensin del alambre estirado?
Ejercicio 2:
Objetivo: Anlisis de fluidos en equilibrio.
Para determinar la densidad ac de un aceite no miscible con el agua se utiliza un tubo en U, que
tiene dos ramas graduadas con sus extremos abiertos. Se introduce el agua en una de las ramas,
y luego, por la misma rama, se introduce el aceite. Luego se mide la posicin de la superficie
libre del agua (esta queda al nivel de graduacin 21,20 cm de la escala de una de las ramas); la
del aceite, en la graduacin 28,80 cm de la otra rama y la de separacin entre los dos lquidos en
la graduacin 12,50cm.
Con los datos anteriores determine la densidad del aceite.
Ejercicio 3:
Objetivo: Anlisis de fluidos en equilibrio. Integracin temtica
En el sistema de la Figura: a) Determine el peso W del bloque que podra soportar el pistn A, si
se aplicaran 5 N sobre el pistn B. El dimetro de A es 22,0(1) cm y el de B es 38,0(5) mm. b)
Si aplicando dicha fuerza se empuja el piston B de manera que se desplace 2,0(1) cm hacia
adentro cunto subir el pistn A? c) Al realizarse la operacin descripta en (b), Variar la
energa del bloque colocado en A?, si su respuesta es s de el valor de la variacin e indique
cmo se produjo. Justifique sus respuestas
A = 22,0(1) cm

5N

Ejercicio 4:

= 38,0(5) mm

Cu rso d e Fs ica I

Se gu nd o Cu a t rime st re de 20 14

25

Un manmetro de mercurio de tubo abierto (en forma de U) tiene su rama izquierda conectada a
un recipiente. (a) Cuando la presin manomtrica P dentro del recipiente es de 0.16 x 10 5 Pa,
cul es la altura de la rama derecha si la altura de la rama izquierda con respecto a la parte
inferior del tubo es de 0.22 m? (b) Cules son las alturas cuando la presin manomtrica es de
0.32 x 105 Pa? En este caso, cunto vale la presin absoluta dentro del recipiente si la presin
atmosfrica PS es de 1.05 atm?

Ejercicio 5:
Objetivo: Ver la aplicabilidad de los modelos fsicos a otras disciplinas.
a) Enuncie el principio de Arqumedes y demuestre que si un slido flota en un lquido, la razn
del volumen total del cuerpo al volumen de la parte sumergida es igual a la razn de la densidad
del lquido a la densidad del slido.
b) Un modelo simplificado de la corteza terrestre consiste en suponer que los continentes flotan
sobre una capa de basalto de densidad 3,0 g/cm 3. Calcule la profundidad media de un continente
en este "mar de basalto" aplicando este modelo y suponiendo que la densidad del agua de mar es
de 1,0 g/cm3, la del granito que forma los continentes es de 2,65 g/cm 3, la profundidad media de
todos los ocanos es de 3 km y la altura media de los continentes sobre el nivel del mar es de 1
km.
Ejercicio 6:
Un bloque cbico de madera de 10 cm de arista flota entre dos capas de aceite y agua, estando
su cara inferior 2 cm por debajo de la superficie de separacin. La densidad del aceite es 0,6
g/cm3. a) Cul es la masa del bloque b)Cul es la presin manomtrica en la cara inferior del
bloque

Ejercicio 7
Calcule la aceleracin inicial de un objeto de 90 kg colgado de 45 globos de He de 1,7 m de
dimetro, una vez que se corta la amarra a la que est sujeto. Desprecie el empuje del aire sobre
el objeto. Considere la densidad del aire como 1,29 kg/m 3. Este problema est basado en un
hecho real, esto es, fue protagonizado por una persona (incluido en lo que ac llamamos el
objeto), tal como se cuenta en el artculo que se incluye al final de esta gua.
Ejercicio 8

Cu rso d e Fs ica I

Se gu nd o Cu a t rime st re de 20 14

26

Objetivo: Desarrollar la capacidad para resolver problemas a partir de la bsqueda de la


informacin necesaria y la aplicacin de los conceptos aprendidos
En los dispositivos esquematizados a continuacin, analice las fuerzas aplicadas sobre cada
parte del sistema, indicando qu agente las ejerce y sobre quin se ejercen las reacciones. La
lectura de la balanza inferior es la misma en los tres casos?. Si su respuesta es S, justifique. Si
su respuesta es NO, cul es la mnima cantidad de datos que debera conocer para determinar
cul ser la lectura de la balanza en cada uno de los casos y cul la lectura del dinammetro?
dinammetro

agua

agua

vidrio

agua
corcho

vidrio
o metal

balanza

cuerda
ideal

balanza

balanza

Indique claramente cul o cules son los sistemas objeto de anlisis en cada caso.
Ejercicio 9:
Determinar la tensin T en la cuerda de una barra homognea que tiene una longitud L=1m y
una masa m=5Kg. La barra es de madera, de densidad = 0,85g/cm, y se encuentra sumergida
hasta la mitad.

Ejercicio 10:
Bloque con plomo
Cuntos metros de plomo (L) habr que agregar a un bloque de madera de las dimensiones
indicadas en la figura, para que flote justo al ras en una pileta de agua?
madera: m = 0,9 g/cm3
plomo: Pb = 11 g/cm3

Cu rso d e Fs ica I

Se gu nd o Cu a t rime st re de 20 14

27

agua: H2O = 1 g/cm3

Ejercicio 11:
Objetivo: Revisar el manejo de la ley de Arqumedes para sistemas que no se pueden
describir con el modelo de partcula.
Un aro de un material desconocido que esta colgado de un dinammetro se sumerge en agua
indicando el dinammetro 1.20 N, si la indicacin del mismo en aire es de 1,50 N, determinar la
densidad de un liquido desconocido si al sumergirlo en este la indicacin del dinammetro es de
1,00 N.
Ejercicio12
Una piedra de 1 kg de masa que cuelga de un dinammetro se encuentra suspendida bajo la
superficie del agua, siendo la indicacin del dinammetro de 7.8 N. (a) Cul es la fuerza de
empuje que ejerce el agua sobre la piedra? (b) Si el recipiente con agua colocado sobre una
balanza pesa 9.8 N, cul ser la indicacin de la balanza cuando la piedra (sin el dinammetro)
est suspendida debajo de la superficie del agua?
Ejercicio 13:
Una corona de oro es pesada a travs de un dinammetro siendo el peso de la misma 6 Nr., si se
la sumerge en agua la indicacin del dinammetro es de 5,5 N., Si la densidad del oro es de 19,8
gr/cm3, La corona es de oro?

Bibliografa
Resnick - Halliday - Krane: Fisica Vol.I 4a.o 5a ed. CECSA
P. Tipler. Fsica Vol 1, cuarta edicin, Revert, 2001
Tipler y Mosca Fsica Vol. 1. Revert, 2005.
Sears Zemansky,Young, novena Ed. Fsica Universitaria Vol. I
Seway y Jewett, Fsica Vol. 1, 2004. Thomson.
Si tiene alguna duda sobre la bibliografa que est usando consulte con los docentes o solicite
algn libro del paol.

Cu rso d e Fs ica I

Se gu nd o Cu a t rime st re de 20 14

28

FISICA I
MODULO II
CLASE 8

Objetivos de la clase:
Estudiar la dinmica de fluidos ideales: Introducir la nocin de flujo estacionario y la
ecuacin de continuidad. Introducir el Teorema de Bernoulli y sus aplicaciones. Introducir
algunas nociones de dinmica de fluidos reales.
Ejercicio 1:
Objetivo: Desarrollar la capacidad para resolver problemas a partir de la bsqueda de la
informacin necesaria y la aplicacin de los conceptos aprendidos. Desarrollo de la
capacidad de diseo.
Un ingeniero debe determinar el caudal que circula por una tubera. A tal fin utiliza un tubo
Venturi. a) Explique en qu consiste. Si no tiene a mano un libro de texto, solicite uno en el
paol y avergelo. b) Si el fluido que circula por la caera fuera voltil, le servira el sistema
de medida presentado en el libro. Explique por que s o por qu no. Si su respuesta fuera
negativa, esquematice un sistema de medida que le permitiera determinar el caudal y explquelo.
c) Qu mediciones debera efectuar y qu ecuaciones podra plantear para determinar la
velocidad con que circula el fluido? d) Qu suposiciones estn implcitas para que las
ecuaciones que utiliza sean realmente vlidas? Explique claramente su razonamiento y el
marco de validez de cualquier ecuacin que utilice.
Ejercicio 2:
Objetivo: Aplicar el teorema de Bernoulli.
La figura muestra un tanque de agua con una vlvula en su parte inferior. Cuando la
vlvula se abre, cul es la mxima altura que puede alcanzar el chorro de agua que emerge del
cao de la derecha? Suponer que h=10 m, L = 2,0 m y = 30.0, y que la seccin normal del
rea en A es mucho mayor que en B (explique cmo emplea esta suposicin)

Cu rso d e Fs ica I

Se gu nd o Cu a t rime st re de 20 14

29

Ejercicio 3:
Objetivo: Entender el funcionamiento del dispositivo llamado sifn
Se usa un sifn para drenar agua de un tanque (o de alguno de los pozos que abundad en las
ciudades, y que se han realizado para hacer los cimientos de una vivienda multifamiliar despus
de alguno de los aguaceros que suelen asolarlas) , como se indica en la figura. El sifn tiene un
dimetro uniforme d (por ejemplo una manguera).
a) encontrar la expresin para el caudal en el extremo inferior del tubo.
b) Existe algn lmite en la altura del punto 2 con relacin a las alturas de los puntos 1 y 3 que
impida el funcionamiento del sifn?
c) Si el lmite se alcanza al querer vaciar el pozo? A qu recurrira para vaciarlo?
Explicite las aproximaciones que ha tenido que realizar para responder a) y b).

Ejercicio 4:
Objetivo: Analizar distintos sistemas de medida de velocidad de fluidos en movimiento
La figura muestra una variante del tubo de Pitot empleada para medir la velocidad v en el seno
de un fluido de densidad . Calcule v en
funcin del desnivel h entre las dos ramas del

v
manmetro y la densidad del fluido
manomtrico.

Pregunte al docente a cargo por el programa que simula un sistema en movimiento en un


fluido y permite realizar medidas con un tubo de Pitot.
Ejercicio 5:

Cu rso d e Fs ica I

Se gu nd o Cu a t rime st re de 20 14

30

En la figura se representa un tubo de Venturi, por el que circula agua. (a) Probar que la
velocidad del fluido en el punto A viene dada por

, donde AA y BB

son las secciones en los puntos A y B, y h es la diferencia de altura del agua en los tubos
verticales. Determinar la velocidad del agua en A para h = 10 cm si el radio de la seccin mayor
es el doble que el de la menor. (b) Calcular el volumen de agua que fluye por unidad de tiempo
a travs de la seccin donde se encuentra el punto B.

Ejercicio 6:
Objetivo: Analizar los lmites de validez del modelo fluido ideal y de las suposiciones y
aproximaciones que conducen a la ecuacin de Bernoulli.
La altura del agua salada ( = 1025 kg/m3) en un depsito cerrado de gran seccin es de 4,8 m.
Un tubo horizontal parte del fondo del depsito, disminuyendo su seccin desde 450 cm 2 hasta
225 cm2, en la forma
indicada en la figura.
El tubo tiene tres prolongaciones verticales A, B y
C, abiertas a la atmsfera.
La presin manomtrica del
aire comprimido contenido
en el depsito es 0,28
kg/cm2 (a cuntos Pascal
equivale?)
a) Si el tubo est abierto en
el punto O, qu altura
alcanza el nivel de agua en
cada uno de los vasos A, B
y C?
b) Explique claramente qu suposiciones y aproximaciones sobre el tipo de flujo y de fluido
realiz para poder determinar analticamente dichas alturas. Suponga, adems, que el intervalo
de tiempo en el que hace la observacin es suficientemente corto de modo que la altura del agua
en el depsito y la presin del aire por encima de la superficie de aqulla se pueden considerar
constantes.
c) Si en lugar de agua el sistema contuviera glicerina sus respuestas al punto a) cambiaran?
Aclare si debera modificar el modelo explicado en b).
Ejercicio 7:
Objetivo: Problema de integracin conceptual
a) Un gran tanque de almacenamiento ubicado a ras de tierra se llena hasta una altura h0. Si se le
hace un orificio a una altura h medida desde el piso (con h < h0), cun lejos del tanque tocar
tierra el chorro? Averige si variara la respuesta en caso de tratarse de un fluido no ideal.

Cu rso d e Fs ica I

Se gu nd o Cu a t rime st re de 20 14

31

b) Si el orificio se fuera achicando el chorro slo se afinara hasta un cierto punto y luego
comenzara a gotear. Puede explicar por qu? De qu depender el tamao de las gotas? Si no
encuentra fcilmente las razones, recurra a su libro de texto o solicite uno en el paol.

Ejercicio 8:
Del depsito A de la figura sale agua continuamente pasando travs de depsito cilndrico B por
el orificio C. El nivel de agua en A est una altura de 12 m sobre el suelo. La altura del orificio
C es de 1.2 m sobre el suelo. El radio del depsito cilndrico B es 10 cm y la del orificio C,4cm.
Calcular:
a) La velocidad del agua que sale por el orificio C.
b) La presin del agua en el punto P del depsito pequeo B
c) La altura h del agua en el manmetro abierto vertical.

Dato: la presin atmosfrica es 101293 Pa.


Ejercicio 9:
La figura muestra como se angosta al caer la corriente de agua que sale de un grifo. El rea de
la seccin transversal Ao es 1.2 cm2 y la de A es de 0.35 cm2. Los dos niveles estn separados
por una distancia vertical h de 45 mm,En que cantidad fluye agua de la llave?

Bibliografa
Resnick - Halliday - Krane: Fisica Vol.I 4a.o 5a ed. CECSA
P. Tipler. Fsica Vol 1, cuarta edicin, Revert, 2001
Tipler y Mosca Fsica Vol. 1. Revert, 2005.
Sears Zemansky,Young, novena Ed. Fsica Universitaria Vol. I
Seway y Jewett, Fsica Vol. 1, 2004. Thomson.

Cu rso d e Fs ica I

Se gu nd o Cu a t rime st re de 20 14

32

FISICA I
MODULO II
CLASE 9
Objetivos
- Analizar a la materia como un sistema de partculas.
- Introducir los conceptos de variables intensivas y extensivas.
- Introducir las nociones de temperatura, energa interna y calor.
- Introducir el Principio cero de la termodinmica.
- Introducir el concepto de dilatacin trmica de slidos.
Nociones introductorias:
Los conceptos introducidos en el mdulo I y en las clases 1 a 6 del mdulo II nos
permiten caracterizar sistemas cuya evolucin temporal queda totalmente definida si se conocen
su estado inicial (determinado por un nmero finito de vectores posicin y vectores velocidad)
y las interacciones que sobre ellos se ejercen (fuerzas y torques exteriores). Cuando se intenta
estudiar sistemas de muchas partculas que no mantienen su forma (lquidos y gases) esta
descripcin resulta compleja (se requeriran del orden de 10 25 coordenadas y velocidades para
describir el estado de un litro de alcohol) y para muchas aplicaciones innecesaria, dado que slo
sera de inters conocer el comportamiento del sistema como un todo, es decir el
comportamiento macroscpico (recordar lo visto para fluidos).
La referencia a la naturaleza microscpica de la materia nos ayuda a entender los
observables (magnitudes medibles) macroscpicos. Estos observables son ms accesibles por su
mayor estabilidad temporal. En el tiempo que se tarda en realizar la medida de la presin en un
lquido a una cierta profundidad los tomos del sistema realizan movimientos extremadamente
rpidos y complejos. Si pensramos en una medida rpida como la obtenida por medio de una
cmara de video (cada cuadro se obtiene en 1/24 de segundo) los tomos de cualquier slido
experimentaran en ese mismo tiempo del orden de 10 millones de vibraciones.
Para relacionar el comportamiento microscpico con el macroscpico se deben utilizar
mtodos estadsticos que permiten calcular los valores medios de las magnitudes dinmicas en
lugar de los valores individuales exactos para cada componente del sistema.
Con lo anterior in mente cul es el objeto de estudio de las clases que nos quedan, es
decir de la Termodinmica?
Vamos a estudiar las consecuencias macroscpicas de los movimientos microscpicos
de los tomos que constituyen los distintos materiales. Nos va a interesar aprender cmo
manipular inteligentemente materiales para obtener formas tiles de energa. Para este ltimo
fin deberemos realizar un anlisis energtico de los sistemas de inters.
Para simplificar el tratamiento en este curso vamos a tratar con sistemas
macroscpicamente homogneos e istropos, elctricamente neutros y qumicamente inertes
que no estn sometidos a interacciones elctricas o magnticas. Nuestros sistemas sern

Cu rso d e Fs ica I

Se gu nd o Cu a t rime st re de 20 14

33

suficientemente grandes como para poder despreciar los efectos de superficie. En este curso,
adems, como una primera aproximacin a la descripcin de la materia vamos a emplear el
modelo de sistema de partculas con el que hemos trabajado anterior mente

Equilibrio Termodinmico
Experimentalmente se observa que en todo sistema termodinmico, S, existen unos
estados privilegiados denominados estados de equilibrio, tales que son estables en el tiempo y
continan sindolo cuando se asla el sistema o cualquier subsistema (parte) del mismo. 1* Esto
implica que el sistema est en:
i) Equilibrio mecnico (visto en el curso).
ii) Equilibrio qumico (las sustancias del sistema no reaccionan qumicamente entre s).
iii) Equilibrio trmico: las diferentes partes del sistema estn todas a la misma temperatura.
Entonces equilibrio termodinmico implica que se cumplan simultneamente los tres
tipos de equilibrio. Todo subsistema de un sistema en equilibrio se encuentra en un estado de
equilibrio.
Variables termodinmicas.
Las variables termodinmicas de un sistema S son los distintos parmetros asociados a
su composicin qumica y a las magnitudes fsicas macroscpicas que lo caracterizan en los
estados de equilibrio.
Parte de estas variables, como son por ejemplo la presin P y el volumen V de un
fluido, la masa, la densidad, la aceleracin de la gravedad, etc. se introducen en otras reas de la
Fsica distintas de la Termodinmica.
Existen algunas variables como son la temperatura T,
introduciremos en el contexto de la Termodinmica.

la entropa S, etc., que

La ecuacin que liga las variables termodinmicas de un sistema se denomina ecuacin


de estado. Todo sistema termodinmico tiene su propia ecuacin de estado, aunque en
ocasiones sta puede resultar difcil de expresar mediante ecuaciones matemticas sencillas. La
ms conocida de estas ecuaciones es la que relaciona la presin, el volumen y la temperatura en
los gases ideales.
Variables intensivas y extensivas.
Las variables termodinmicas aditivas, en el sentido de que su valor en un sistema es
siempre la suma de sus valores en cada uno de los subsistemas en que pueda descomponerse el
sistema en estudio se denominan variables extensivas. Por ejemplo el volumen, la masa, los
nmeros de moles de las especies qumicas que componen el sistema, etc.

Tenga en cuenta que la definicin de estado de equilibrio no implica que estos slo existan en los
sistemas aislados. nicamente se exige que estos estados sigan siendo estables en el tiempo cuando se
asla el sistema o cualquier subsistema del mismo.
Cu rso d e Fs ica I

Se gu nd o Cu a t rime st re de 20 14

34

Las variables definidas localmente en cada punto del sistema, tales que en cada
subsistema homogneo tienen el mismo valor, se denominan variables intensivas. Por ejemplo
la presin, la densidad, etc.
Las variables extensivas pueden expresarse en funcin de sus valores por unidad de
masa (denominndose entonces variables especficas) o bien en funcin de sus valores por mol,
(denominndose entonces variables molares). Tanto las variables especficas como las
variables molares son variables intensivas. Es decir el volumen o la masa de un sistema
depende de la cantidad de materia que lo constituye, pero la densidad o el volumen especfico
slo dependen de la calidad (la clase) de materia que lo constituye (hierro, agua, DNA)
*Recurdese que un mol de una sustancia contiene un nmero de tomos o molculas
igual al nmero de Avogadro (NA = 6,023 x 1023).
Si V es el volumen de un sistema homogneo formado por N moles de una sustancia
V
pura de masa m, el volumen molar es:
N
y el volumen especifico es : '

V
m

Si definimos que M es su peso molecular, la masa puede expresarse como: m = N M


Entonces, y ' estn relacionadas por: M ' .
Ligaduras.
Todo sistema S se encuentra en general sometido a un conjunto de condiciones
restrictivas para las variables termodinmicas. Esas condiciones son impuestas por la naturaleza
de las paredes que separan a S del medio exterior, por la naturaleza de las paredes que separan a
los distintos subsistemas de S entre s, o por el tipo de interaccin que el medio exterior puede
tener con S. Estas condiciones restrictivas se denominan ligaduras, y se llaman externas si
las impone el medio exterior o las paredes que lo separan de S. Las ligaduras se denominan
internas en el caso de que las impongan las paredes que separan entre s a los distintos
subsistemas de S.
En particular, se dice que el sistema es cerrado cuando S est rodeado de una pared
impermeable2. Esta le impide intercambiar materia con el medio exterior. En caso contrario se
dice que el sistema es abierto.
Se dice que el sistema es aislado cuando S est rodeado de una pared que no permite
ningn tipo de intercambio con el medio exterior.
Escala s termomtricas
En todo cuerpo material la variacin de la temperatura va acompaada de la
correspondiente variacin de otras propiedades medibles, de modo que a cada valor de aqulla
le corresponde un solo valor de sta. Tal es el caso de la longitud de una varilla metlica, de la
resistencia elctrica de un metal, de la presin de un gas, del volumen de un lquido, etc. Estas

Ntese que pueden existir paredes semipermeables, que solo permiten pasar a travs de ellas a ciertas
especies qumicas de entre todas las que componen el sistema, por lo que no evitan completamente su
contacto respecto al intercambio de materia con el medio exterior.
Cu rso d e Fs ica I

Se gu nd o Cu a t rime st re de 20 14

35

magnitudes cuya variacin est ligada a la de la temperatura se denominan propiedades


termomtricas, porque pueden ser empleadas en la construccin de termmetros.
Para definir una escala de temperaturas es necesario elegir una propiedad termomtrica
que rena las siguientes condiciones:
a.
La expresin matemtica de la relacin entre la propiedad y la
temperatura debe ser conocida.
b.
La propiedad termomtrica debe ser lo bastante sensible a las
variaciones de temperatura como para poder detectar, con una precisin aceptable,
pequeos cambios trmicos.
c.
El rango de temperatura accesible debe ser suficientemente grande.

Escala Celsius

Una vez que la propiedad termomtrica ha sido elegida, la elaboracin de una escala
termomtrica o de temperaturas lleva consigo, al menos, dos operaciones; por una parte, la
determinacin de los puntos fijos o temperaturas de referencia que permanecen constantes en la
naturaleza y, por otra, la divisin del intervalo de temperaturas correspondiente a tales puntos
fijos en unidades o grados.
El cientfico sueco Anders Celsius (1701-1744) construy por primera vez la escala
termomtrica que lleva su nombre. Eligi como puntos fijos el de fusin del hielo y el de
ebullicin del agua, tras advertir que las temperaturas a las que se verificaban tales cambios de
estado eran constantes a la presin atmosfrica. Asign al primero el valor 0 y al segundo el
valor 100, con lo cual fij el valor del grado centgrado o grado Celsius (C) como la centsima
parte del intervalo de temperatura comprendido entre esos dos puntos fijos.

Escala Fahrenheit

En los pases anglosajones se pueden encontrar an termmetros graduados en grado


Fahrenheit (F). La escala Fahrenheit difiere de la Celsius tanto en los valores asignados a los
puntos fijos, como en el tamao de los grados. As al primer punto fijo se le atribuye el valor 32
y al segundo el valor 212. Para pasar de una a otra escala es preciso emplear la ecuacin:
t(F) = 1,8 t(C) + 32
donde t(F) representa la temperatura expresada en grados Fahrenheit y t(C) la
expresada en grados Celsius o centgrados.

Escala Kelvin

La escala de temperaturas adoptada por el SI es la llamada escala absoluta o Kelvin. En


ella el tamao entre grado y grado es el mismo que en la Celsius, pero el cero de la escala se fija
en el - 273,16 C. Este punto llamado cero absoluto de temperaturas es tal que a dicha
temperatura desaparece la agitacin molecular, por lo que, segn el significado que la teora
cintica atribuye a la magnitud temperatura, no tiene sentido hablar de valores inferiores a l. El
cero absoluto constituye un lmite inferior natural de temperaturas, lo que hace que en la escala
Kelvin no existan temperaturas bajo cero (negativas). La relacin con la escala centgrada viene
dada por la ecuacin:
T(K) = t(C) + 273,16

Cu rso d e Fs ica I

Se gu nd o Cu a t rime st re de 20 14

36

siendo T(K) la temperatura expresada en grados Kelvin o simplemente en Kelvin.

Expansin trmica de slidos


La expansin trmica de un objeto es consecuencia de los cambios que se producen en
la separacin promedio entre los tomos o las molculas que lo constituyen. Si la expansin
trmica de un objeto es suficientemente pequea comparada con las dimensiones iniciales del
objeto, el cambio que se produce en cualquiera de sus dimensiones es proporcional al cambio
de la temperatura.
Lf = Li (1 + ( Tf- Ti ) )
donde Li y Lf representan la longitud inicial y final, T i y Tf la temperatura inicial y
final y es el coeficiente promedio de expansin lineal para un material determinado y tiene
unidades de (C)-1.
Si transferimos calor a un alambre tenemos la ecuacin anterior, si la dilatacin se
realiza en dos dimensiones, o sea una chapa la ecuacin se convierte en:
Af = Ai (1 + ( Tf- Ti ))
donde es el coeficiente de dilatacin superficial y es igual a 2, Af es la superficie
final, Ai es la superficie inicial, Tf es la temperatura final y Ti es la temperatura inicial.
Si la transferencia de calor se realiza sobre un cuerpo de volumen determinado la
ecuacin queda:
Vf = Vi ( 1 + (Tf Ti))
donde es el coeficiente de dilatacin cbica del material que es igual a 3, donde Vf es el
volumen final, Vi es el volumen inicial, Tf es la temperatura final y Ti es la temperatura inicial.
Tabla de coeficientes de dilatacin lineal para distintos materiales:
Alumnio
Cobre
Hierro
Oro
Plata
Platino
Plomo
Vidrio
Estao

23 * 10-6 1/C
17 * 10-6 1/C
12 * 10-6 1/C
13 * 10-6 1/C
17 * 10-6 1/C
9 * 10-6 1/C
29 * 10-6 1/C
9 * 10-6 1/C
27 * 10-6 1/C

Ejercicio 1: Determinar la temperatura en Kelvin y Centgrado de: 23F, 120F, 50F, 315F.
Ejercicio 2: Cul ser la temperatura en la que el grado centgrado coincide con el grado
Fahrenheit?
Ejercicio 3: Qu es mayor, un grado centgrado, un grado Fahrenheit o un Kelvin?

Cu rso d e Fs ica I

Se gu nd o Cu a t rime st re de 20 14

37

Ejercicio 4: Determinar la temperatura en grados Fahrenheit de: 110C, 23 K, 150C, 300 K,


23C.
Ejercicio 5: Determinar la temperatura en escala absoluta o kelvin de: 45F, 27C, 500C,
212F, 32C, 400F.
Ejercicio 6: a) Dos barras A y B de la misma longitud inicial, sufren la misma elevacin de
temperatura. Podrn ser diferentes las dilataciones de estas barras? Explicar. b) Dos barras A y
B del mismo material, experimentan la misma elevacin de temperatura. Podrn ser diferentes
las dilataciones de estas barras? Explicar.
Ejercicio 7: Una barra de metal de 5 metros de longitud a 18C, cuando se calienta a 80 C
sufre un incremento de longitud de 0,0015 metros.Cul es el coeficiente de dilatacin lineal del
metal?
Ejercicio 8: La longitud de un puente de hierro es de 1000 metros. Tiene un extremo fijo y el
otro libre sobre rodillos.Cunto se desplaza ante un cambio de temperatura de 10C a 45?
Ejercicio 9: Se colocan dos tramos prefabricados de hormign en un puente de 250 m de
longitud, de manera que no queda espacio para una posible expansin. Si se produce un
aumento de temperatura de 20C. Cunto mide la altura y a la que se levantan ambos tramos
al deformarse?
y
250 m

Ejercicio 10: Una vidriera de 10 metros cuadrados sufre en verano una elevacin de
temperatura de 25C, Cul ser la superficie final despus de sufrir esa elevacin de
temperatura?
Ejercicio 11: Un objeto de vidrio a 20C tiene un volumen inicial de 700 centmetros cbicos.
Cul ser su volumen a una temperatura de 100C.

Bibliografa
R. Resnick and D. Halliday. Fsica. Parte 1. 3ra. Edicin. Captulo 22 tem 1a 7.
P. Tipler. Fsica. Captulo 18 tem 1 a 3.
F. W. Sears. Mecnica, Calor y Sonido. Captulo 19 22 tem 1 y 22 tem 6.
En las nuevas ediciones, buscar los temas: Ley cero de la termodinmica. 1er Principio de la
Termodinmica Calor y Trabajo.

Cu rso d e Fs ica I

Se gu nd o Cu a t rime st re de 20 14

38

FISICA I
MODULO II
CLASE 10
Objetivos
- Introducir y reforzar los conceptos trabajo (rea bajo la curva en un diagrama PV) y calor
como procesos de transferencia de energa.
- Introducir mecanismos de transferencia de energa en forma de calor.
- Reforzar el manejo del Primer principio de la termodinmica y aplicarlo a distintos sistemas
fsicos.
Procesos de transferencia:
El calor se puede propagar a travs de los siguientes mecanismos:
Radiacin
La radiacin en forma de ondas electromagnticas es un mecanismo de transmisin de calor.
Todo cuerpo por encima de cero grado Kelvin (0K = -273 C) emite radiacin en forma de
ondas electromagnticas El ritmo mediante el cual un cuerpo radia energa trmica es
proporcional al rea del cuerpo y a la cuarta potencia de su temperatura absoluta. La ley de
Stefan-Boltzmann describe esta forma de transmisin de calor
I = e A T4. La cantidad I
es la energa total transmitida, por segundo, por una superficie A, a la temperatura T; e es una
constante denominada emisividad, cuyo valor vara entre 0 y 1, que depende de la naturaleza de
la superficie emisora y = 0.567 10-4 erg cm-2 grado-4 s-1, es la constante de Stefan-Boltzmann.
Tambin consideraremos la absorcin de radiacin trmica por una superficie. En este proceso,
se elimina energa de la radiacin incidente, a travs de su accin sobre las cargas elctricas. Es
interesante la relacin que existe entre la eficiencia de la superficie como emisor, medida por e,
y su eficiencia como absorbente. Esta la mide una constante, llamada absortividad, a, que se
define como la relacin entre la energa trmica total absorbida por la superficie y la energa
trmica total que incide sobre ella. La relacin entre e y a fue descubierta por Kirchhoff y es
simplemente,
e=a
Tabla de valores de absortividad para la radiacin solar y emisividad de radiacin terrestre.
Materiales de superficies distintas
Pelcula de aluminio brillante
Pelcula de aluminio no brillante
Pintura de aluminio
Pintura blanca
Chapa galvanizada
Hormign
Ladrillo o tejas

Cu rso d e Fs ica I

a
0.05
0.1
0.40
0.20
0.89
0.70
0.70

Se gu nd o Cu a t rime st re de 20 14

e
0.05
0.12
0.90
0.90
0.90
0.94
0.90

39

Terminacin asfltica

0.92

0.90

Conveccin
En este mecanismo el calor es transmitido a travs de un transporte directo de masa. Esta forma
se da en fluidos, (lquidos y gases) porque implica el desplazamiento de una masa determinada
de la sustancia de una zona donde la misma se encuentre a mayor temperatura a otra donde est
a menor temperatura. El Flujo calrico q recibido es proporcional a la diferencia de
temperatura ta ts siendo ta = temperatura ambiente y t s = temperatura superficial; y a un
coeficiente de conveccin C expresado en W/ m2 C.
Valores del coeficiente de conveccin C para el aire en calma.
C = 1,16 W/ m2.C
C = 6,96 W/ m2.C
C = 4,64 W/ m2.C

Conveccin pequea
Conveccin muy fuerte
Conveccin intermedia

Por ejemplo la energa trmica del aire caliente se transporta del suelo al techo
junto con la masa de aire caliente.
La fuerza de gravedad separa los fluidos en reposo de acuerdo con su densidad: los fluidos ms
densos en los niveles ms bajos mientras que los ms livianos o menos densos arriba.
Dado que la conveccin implica desplazamientos de fluidos, es en general un proceso
complicado, porque el movimiento de los fluidos lo es. Los procesos convectivos son los
responsables de los fenmenos atmosfricos e intervienen en las corrientes marinas, en la escala
planetaria. En el orden domstico, tienen especial relevancia en la calefaccin y ventilacin
natural.
Conduccin
El proceso de conduccin del calor es simplemente la propagacin de la agitacin trmica por
difusin. Si se calienta una barra de algn material, los tomos del extremo caliente se agitan
fuertemente chocando con los tomos vecinos, de esta manera los tomos que se agitan ms
fuertemente ceden parte de su energa a los que lo hacen con menos intensidad. El calor difunde
por conduccin de las zonas ms calientes a las zonas mas fras. No se da en forma natural el
proceso inverso.
Veremos los procesos de conduccin en estado estacionario, es decir aqullos en los que la
distribucin de temperatura de un cuerpo no cambia a lo largo del tiempo. La funcin que rige
este proceso es la llamada ley de Fourier:
H Q / t k A T / x

El termino Q/t se denomina flujo de energa trmica o corriente trmica, k representa la


conductividad trmica y depende del material, A es la seccin transversal, T es la variacin de
temperatura entre los extremos de la barra y x es la longitud de la barra o el espesor de la
placa segn sea el caso.

Material
Aire a 27 C
hielo
Cu rso d e Fs ica I

k (Watt/m.K)
0,026
0,592
Se gu nd o Cu a t rime st re de 20 14

40

Agua a 27 C
Aluminio
Cobre
Oro
Hierro
Acero
Roble
Pino blanco
Hormign
Vidrio

0,609
237,00
401,00
318,00
80,40
46,00
0,15
0,11
0,19 a 1,3
0,7 a 0,9

De la expresin de Fourier podemos despejar T = H (x / k. A)


o bien T = H . R
donde R = x/k.A.
Esta expresin es similar a la que existe en los circuitos elctricos de corriente continua entre la
corriente elctrica (I) que circula por una resistencia (R) y la diferencia de potencial que existe
entre sus extremos (V), de acuerdo a la expresin conocida como Ley de Ohm: V = I. R.
Los materiales conductores se pueden colocar en serie (uno a continuacin de otro), cada uno
con resistencia Rn. En este caso, la resistencia equivalente R eq es la suma de todas las
resistencias en serie: Req = R1 +R2 +R3 ......... + Rn .
Otra forma de colocarlos es en paralelo, por ej. cuando consideramos una pared que contiene
una ventana y ambas se encuentran separando el interior del exterior a distintas temperaturas.
En este caso, las caras interiores del vidrio de la ventana y de la pared estn a la misma
temperatura, mientras que las caras que dan al exterior estn a otra temperatura.
La resistencia equivalente (Req) para el caso paralelo viene dada por expresin:
1/Req = 1/R1 + 1/R2 + 1/R3 +.........+ 1/Rn
Ejercicio 1
Una plancha aisladora trmica tiene 100 cm2 de rea y 2 cm de espesor y su conductividad
trmica es 2x10-4 cal/(s C cm). Si la diferencia de temperaturas entre caras opuestas se
mantiene a 100 C, calcule la cantidad de energa que atraviesa la plancha en un da.
Ejercicio 2
Una barra, de seccin transversal A = 2 cm2, consta de dos partes: una de cobre de longitud 30
cm y otra de acero de longitud 20 cm. La superficie lateral de la barra est trmicamente aislada.
El extremo de acero se encuentra en una mezcla de hielo en fusin y agua y el extremo de cobre
en un bao de vapor. Cul es la temperatura en la unin de los dos metales?
Ejercicio 3
a) Calcule la rapidez con que el cuerpo emite calor a travs de la ropa de un esquiador,
si se conocen los siguientes datos: la superficie corporal mide 1.80m 2 y el espesor de la ropa es
de 1.2cm; la temperatura de la piel es 33C, en tanto que la superficie externa de la ropa tiene
una temperatura de 1.0C; la conductividad trmica de la ropa es 0.040W/m.K. b) Cmo
cambiara la respuesta si, despus de una cada, la ropa del esquiador se empapara con agua?
Suponga que la conductividad trmica del agua es 0.60 W/m.K
Ejercicio 4

Cu rso d e Fs ica I

Se gu nd o Cu a t rime st re de 20 14

41

A una barra de cobre de 1 kg que est a presin atmosfrica y a 20C, se le aumenta la


temperatura a 80C ponindola en contacto con una fuente trmica.
a)Qu modificaciones macroscpicas se observarn en la barra? b) Cul sera el modelo
microscpico adecuado para analizar la situacin? c) Qu trabajo realiza el cobre sobre la
atmsfera durante el proceso de calentamiento? d) Qu cantidad de energa en forma de calor
se transfiere de la fuente trmica al cobre? e) Cul es el incremento de la energa interna del
cobre? f) Si los extremos de la barra estuvieran empotrados en una estructura de hormign
variaran sus respuestas anteriores? Explique su razonamiento.
Datos del Cu que podra necesitar: Coeficiente de dilatacin lineal: 14 10 -6 1/C Densidad: 8.02
103 kg./m3. Calor especifico: 387 J/kg.k
Ejercicio 5
Un sistema fsico recorre el ciclo de la figura, del estado a al b y de regreso al a. El
valor absoluto de la transferencia de calor durante un ciclo
p
es de 7200 J. a) El sistema absorbe o desprende calor
cuando recorre el ciclo en la direccin indicada en la figura?
b
b) Qu trabajo W efecta el sistema en un ciclo? c) Si el
sistema recorre el ciclo en direccin antihoraria, absorbe o
desprende calor en un ciclo? Qu magnitud tiene el calor
absorbido o desprendido en un ciclo antihorario?
Ejercicio 6

Cuando se hace pasar un sistema del estado i al estado f,


V
siguiendo la trayectoria i-a-f en el plano PV, se encuentra O
que los procesos calor (Qif) y trabajo (wif) generan
transferencias de energa con el entorno de valor absoluto 50
cal y 20 cal, respectivamente.
a) Analice, de acuerdo con el primer principio de la
termodinmica, si dichos procesos aumentan o disminuyen la
energa interna del sistema.
b) Cul sera la energa transferida por el proceso trabajo si se
pasara del estado i al f siguiendo la trayectoria i-b-f (en el
plano PV) y se determinara experimentalmente que la energa transferida por el proceso calor es
de Qibf = 36 cal?el proceso trabajo en este caso transferira energa desde o hacia el sistema
(disminuira o aumentara la energa interna del mismo)?
c) Si luego se pasa del estado f al i siguiendo una nueva trayectoria, tal que a travs del proceso
trabajo se transfieren 13 cal entre el sistema y el entorno, cul sera la energa transferida en
forma de calor en dicha trayectoria?
En este caso, el proceso calor transfiere energa desde o hacia el sistema? d) Si la energa
interna del sistema en el estado i fuera Ui = 10 cal, calcule la energa interna que tendra el
estado f, Uf.
e) Si la energa interna del estado b fuera Ub = 22 cal, calcule la energa que se transferira al
sistema por el proceso calor en los trayectos i-b y b-f.
f) La energa transferida por cada uno de los procesos, Q y W, al pasar del estado i al f siguiendo
la trayectoria i-a-f ser igual o distinta a la transferida por cada uno de dichos procesos cuando
el sistema pasa del estado i al f siguiendo la trayectoria i-b-f? Explique su razonamiento.
g) La variacin de la energa interna del sistema al pasar del estado i al f siguiendo la trayectoria
i-a-f ser igual o distinta a la experimentada por el sistema al pasar del estado i al f siguiendo
la trayectoria i-b-f? Explique su razonamiento.

Cu rso d e Fs ica I

Se gu nd o Cu a t rime st re de 20 14

42

Ejercicio 7
(a) Determinar el trabajo realizado sobre un fluido que se expande desde i hasta f como se indica
en la figura. (b) Cunto trabajo se realiza sobre el fluido si se lo comprime desde f hasta i a lo
largo de la misma trayectoria?

P (x 106 Pa)
6

4
f

V (m3)

Ejercicio 8
En cul de las trayectorias entre el estado i y el estado f que se muestran en figura siguiente se
realiza el mximo trabajo en el gas?

Ejercicio 9
Se lija vigorosamente con una lijadora elctrica la superficie externa de un recipiente metlico,
que contiene un fluido en su interior. Un termmetro est en contacto con el fluido a travs de
un pequeo orificio y se observa que la temperatura va aumentando con el tiempo de lijado.
Explicar los procesos energticos que tienen lugar si se toma como sistema de estudio:
a) Slo el fluido. b) Slo el recipiente. c) El recipiente y el fluido. d) El recipiente, el fluido y la
lijadora. e) El recipiente, el fluido, la lijadora y el medio ambiente.
Aclarar, para cada sistema estudiado, cul o cules son los sistemas con los que interacciona
(medio exterior).
Ejercicio 10

Cu rso d e Fs ica I

Se gu nd o Cu a t rime st re de 20 14

43

La cascada del ngel, en Venezuela, es la cada de agua ms alta del mundo con una
profundidad de 975 m. a) Suponiendo que se pudiera despreciar la friccin con el aire, que el
salto partiera del reposo, que la superficie sobre la que choca estuviera en reposo y que luego
del choque el agua quedara en reposo cul sera la velocidad del agua justo antes de alcanzar el
ro que corre abajo? b) Suponiendo que el sistema pudiera considerarse aislado, cul sera la
diferencia entre la temperatura del agua en la parte superior y la del agua del ro debajo de la
cascada? c) Analice la validez de las suposiciones hechas en a) y b).
Ejercicio 11
El calor especfico (por qu piensa que se introduce el concepto calor especfico en reemplazo
de la capacidad calorfica) de un fluido se puede medir con el auxilio de un calormetro de flujo
como el esquematizado en la figura.
El fluido atraviesa el calormetro en flujo estacionario, de forma tal que la masa por unidad de
tiempo que lo atraviesa vm es constante. Penetrando a una temperatura Ti el fluido pasa por una
regin donde funciona un calentador elctrico de potencia P constante y emerge con temperatura
Tf. Si en una experiencia con benceno se han medido los valores, v m = 5,0(1) g/s, P = 200(2) W,
Ti = 15,5(1) C y Tf = 38,8(1) C. Determine el calor especifico del benceno.

Bibliografa
Resnick - Halliday - Krane: Fisica Vol.I 4a.o 5a ed. CECSA
P. Tipler. Fsica Vol 1, cuarta edicin, Revert, 2001
Tipler y Mosca Fsica Vol. 1. Revert, 2005.
Sears Zemansky,Young, novena Ed. Fsica Universitaria Vol. I
Seway y Jewett, Fsica Vol. 1, 2004. Thomson.
Si tiene alguna duda sobre la bibliografa que est usando consulte con los docentes o solicite
algn libro del paol.

Cu rso d e Fs ica I

Se gu nd o Cu a t rime st re de 20 14

44

FISICA I
MODULO II
CLASE 11
Objetivos:
- Introducir el concepto de gas ideal
- Aplicar el Primer principio de la termodinmica a sistemas que pueden modelarse como
gases ideales.
- Estudiar distintos ciclos termodinmicos
- Introducir el concepto de rendimiento de un ciclo termodinmico
.
Problema 1:
Un mol de un gas ocupa un volumen de 10 litros a 1 atm. Cul es su temperatura?
El recipiente que contiene el gas est provisto de un pistn mvil y el gas se pone en contacto
con una fuente trmica que est a una temperatura mayor que la determinada en a). Al alcanzar
el sistema el equilibrio el volumen del gas es de 20 litros. Cul es su temperatura en grados
Kelvin?, y en grados Celsius?
El pistn se fija y se le transfiere al gas energa en forma de calor hasta que su temperatura
alcanza los 350 K. Cul es su presin?
Problema 2:
En un termmetro de gas a volumen constante, la presin a 20C es 0.980 atm. (a) Cul es la
presin a 45.0C? (b) Cul es la temperatura cuando la presin es de 0.500 atm?
Problema 3:
Las dimensiones del auditorio de un colegio son 10 m x 20 m x 30 m. (a) Cuntas molculas
de aire habr en el auditorio si la temperatura es de 20C y la presin es de 101 kPa? (b) Si la
temperatura se incrementa en 5C, permaneciendo constante la presin, cuntos moles de aire
salen de la habitacin?
Problema 4:
Un tanque de 3 litros contiene aire a 3 atm y 20C. El tanque se sella y enfra hasta que la
presin es de 1 atm. a) Qu temperatura tiene ahora el gas en grados Celsius? Suponga que el
volumen del tanque es constante. b) Si la temperatura se mantiene en el valor determinado en la
parte (a) y el gas se comprime, qu volumen tendr cuando la presin vuelva a ser de 3 atm?
Problema 5:

Cu rso d e Fs ica I

Se gu nd o Cu a t rime st re de 20 14

45

En un recipiente cerrado de tres litros de capacidad hay nitrgeno a 27 C y a 3 atm. de presin.


El recipiente se pone en contacto con una fuente trmica. Despus que el sistema llega al estado
de equilibrio, la presin dentro del recipiente aument alcanzando 25 atm. Determine la energa
transferida al nitrgeno en forma de calor y la temperatura final del mismo.
Problema 6:
Se tienen 0.2 moles de un gas encerrados en un cilindro con un mbolo en su parte superior,
estando el sistema en equilibrio. El mbolo tiene una masa de 8 kg y un rea de 5 cm 2, y se
puede mover sin rozamiento hacia arriba o hacia abajo. El cilindro se pone en contacto con una
fuente de calor, de modo que la temperatura del gas se incrementa lentamente desde 20C hasta
30C. Calcular el trabajo realizado sobre el gas en este proceso
Problema 7:
10 g de argn, (PM40 g /mol) se hallan inicialmente a 3 atm y 300 K. El gas sufre una
transformacin hasta llegar a 1 atm y 200 K. Hallar el trabajo, el calor y la variacin de energa
interna para las siguientes transformaciones que llevan el gas desde su estado inicial a su estado
final: a) presin constante seguida de volumen constante, b) volumen constante seguida de
presin constante, c) temperatura constante, volumen constante, d) volumen constante,
temperatura constante. Para cada caso grafique en un diagrama P-V las caractersticas del
proceso.
Problema 8:
Una muestra de 2 moles de un gas ideal con =Cp/Cv = 1,40 se expande lentamente de manera
adiabtica desde una presin de 5 atm y un volumen de 12 litros hasta un volumen final de 30
litros. (a) Cul es la presin final del gas? (b) Cules son las temperaturas inicial y final del
gas? (c) Calcular el calor, el trabajo y la variacin de energa interna ocurridos durante el
proceso.
Problema 9:
Un cilindro de volumen total 4 litros contiene 0,2 moles de un gas perfecto, (constante =
Cp/Cv =1,5) a 300 K. El cilindro se halla aislado trmicamente del exterior y est provisto de
un mbolo perfectamente ajustado y sin roce. Inicialmente el gas ocupa un volumen de un litro.
Se permite al gas expandirse hasta ocupar todo el volumen del cilindro.
a) Si la expansin se realiza elevando lentamente el mbolo, calcule: i) la temperatura y presin
finales, ii) el trabajo realizado (indique si fue realizado por o sobre" el sistema), iii) la
energa transferida en forma de calor (indique si se transfiere hacia o desde el sistema) y
iv) la variacin de energa interna.
b) Si la expansin se realizara abriendo una vlvula, calcule las mismas magnitudes que en a).
Problema 10:
Dos moles de un gas perfecto, para el cual Cv = 3 cal/mol C, efectan el ciclo a-b-c-a de la
figura. El proceso b-c es una expansin isotrmica.
a) Calcule el trabajo realizado por el gas en cada una de las etapas del ciclo.
b) Halle la energa transferida en forma de
calor y la variacin de energa interna, en
cada una de las etapas del ciclo
c) Defina rendimiento de un ciclo trmico y
calclelo para este caso.
Cu rso d e Fs ica I

Se gu nd o Cu a t rime st re de 20 14
8,4

46
16,8

Problema 11:
Se conduce una muestra de 4 litros de un gas diatmico ideal confinada en un cilindro a travs
de un ciclo cerrado. El gas se encuentra inicialmente a 1 atm y 300 K. En primer lugar, se
triplica su presin a volumen constante. Luego se expande adiabticamente el gas hasta su
presin original. Finalmente, se comprime el gas isobricamente hasta su volumen original. (a)
Dibujar un diagrama P-V de este ciclo. (b) Determinar el volumen del gas al final de la
expansin adiabtica. (c) Hallar la temperatura del gas al comienzo de la expansin adiabtica.
(d) Calcular el trabajo neto realizado por y sobre el gas en este ciclo.

Bibliografa
Resnick - Halliday - Krane: Fisica Vol.I 4a.o 5a ed. CECSA
P. Tipler. Fsica Vol 1, cuarta edicin, Revert, 2001
Tipler y Mosca Fsica Vol. 1. Revert, 2005.
Sears Zemansky,Young, novena Ed. Fsica Universitaria Vol. I
Seway y Jewett, Fsica Vol. 1, 2004. Thomson.
Si tiene alguna duda sobre la bibliografa que est usando consulte con los docentes o solicite
algn libro del paol.

Cu rso d e Fs ica I

Se gu nd o Cu a t rime st re de 20 14

47

FISICA I
MODULO II
CLASE 12
Objetivos:

Estudiar procesos de transferencia de energa entre las partes de un sistema


termodinmicamente aislado (rodeado por paredes adiabticas) (a ste tpico se lo suele
denominar en los libros de texto: Calorimetra).
Estudiar transformaciones de fase.
Introducir el modelo de gas real.

Problema 1:
Un gramo de agua (1 cm3) se convierte en 1671 cm3 de vapor cuando hierve a presin de 1 atm.
Qu trabajo realiza el sistema y cul es la variacin de su energa interna durante dicho
proceso?
Lv = 539 cal/gr (2250 J/g)
Problema 2:
Una olla de cobre de 0,500 kg contiene 0,170 kg de agua a 20C. Un bloque de hierro de 0,250
kg a 85C se coloca en su interior. Calcule la temperatura final suponiendo que no hay
transferencia de energa en forma de calor con el entorno. (c cu = 390 J/kg.K; cFe = 470 J/kg.K)
Problema 3:
Qu cantidad de calor es necesaria para elevar la temperatura de 3 kg de cobre en 20C? y en
20K? Calor especifico del Cobre = 0.386 KJ/(kg.K)
Problema 4:
Un cuerpo A posee una masa doble que la de otro cuerpo B. Su calor especfico es tambin el
doble. Si ambos se le suministra la misma cantidad de calor, Qu relacin existe entre los
cambios de experimentados por sus respectivas temperaturas?
Problema 5:
Un tcnico de laboratorio pone una muestra de 0,085 kg de un material desconocido que est a
100 C en un calormetro cuyo recipiente, inicialmente a 19C, est hecho con 0,150 kg de cobre
y contiene 0,200 kg de agua. La temperatura final del calormetro es de 26,1 C. Calcule el calor
especfico de la muestra.
Problema 6:

Cu rso d e Fs ica I

Se gu nd o Cu a t rime st re de 20 14

48

Para medir el calor especfico del plomo se calientan 600g de perdigones de este metal a 100C
y se colocan en un calormetro de aluminio de 200g de masa que contiene 500g de agua
inicialmente a 17.3C. El calor especifico del aluminio del calormetro es 0.9 kJ/kg. La
temperatura final del sistema es 20C. Cul es el calor especfico del plomo?

Problema 7:
Un vaso aislado con masa despreciable contiene 0,250 kg de agua a 75C. Cuntos kg de hielo
a -20C deben colocarse en el agua para que la temperatura final del sistema sea 30C? Datos:
Lf, hielo = 80 cal/gr, chielo = 0,55 cal/gC.
Problema 8:
Dentro de un recipiente de paredes adiabticas (calormetro) que contiene 10 g de agua y 10 g
de hielo se introduce una masa de plomo a 200 C.
a) Si la masa del plomo fuera de 100 g, calcule la temperatura final que adquirira el sistema y
la cantidad de hielo que se fundira. Graficar en forma cualitativa la variacin de la
temperatura en funcin del tiempo para los sistemas agua, hielo, plomo y calormetro.
b) Idem si la masa de plomo introducida fuera de 200 g.
c) Qu cantidad de masa de plomo debera introducirse en el calormetro para convertir en
vapor a la mitad del lquido que est dentro del calormetro?
Lf,hielo = 80 cal/g; Lv,agua = 540 cal/g (a presin atmosfrica); cpb = 0,031 cal/g C
Problema 9:
Una bala de plomo de 10 g de masa y una temperatura de 30C que viaja a una velocidad de 250
m/s se incrusta en un gran bloque de hielo que flota en equilibrio trmico con el agua de un
lago. Calcular la masa de hielo que se habr fundido una vez alcanzado el equilibrio (despreciar
la interaccin con la atmsfera). [Calor especfico del plomo cp Pb = 128 J/(kg C)]
Problema 10:
Un recipiente con paredes trmicamente aisladas contiene 2,4 kg de agua y 0,45 kg de hielo,
todo a 0C. El tubo de salida de una caldera en la que hierve agua a presin atmosfrica se
inserta en el agua del recipiente. Cuntos gramos de vapor deben condensarse dentro del
recipiente (que tambin est a presin atmosfrica) para elevar la temperatura del sistema a
28C? Despreciar el calor transferido al recipiente.
L v = 539 cal/g
Problema 11:
Un mol de cierto gas ideal ocupa un volumen de 24 litros en el interior de un cilindro, en
contacto diatrmico con una mezcla en equilibrio de agua y hielo (ver figura). A partir de un
cierto instante A comienza a bajarse lentamente un pistn (sin roce apreciable con las paredes)
de modo que el gas se comprime hasta alcanzar un estado B en que el volumen es VB = VA/4.
(a) Calcular la masa de hielo que se funde a lo largo del proceso y el cambio en la energa
interna del gas ideal. (b) Si el pistn se comprime en forma brusca hasta que el volumen es VB,
y luego se deja fijo un tiempo suficiente, alcanza el gas el mismo estado final B? Cul habr

Cu rso d e Fs ica I

Se gu nd o Cu a t rime st re de 20 14

49

sido en este caso el cambio en su energa interna? En qu difiere este proceso del considerado
en (a)?

Problema 12:
Dos cilindros transparentes provistos de respectivos pistones contienen uno un gas real y el otro
un gas ideal. Si ambos cilindros estn en contacto con una fuente trmica. Dispone de los
medios para determinar cul contiene gas real y cul contiene gas ideal? Explique su
razonamiento.
Problema 13:
La ecuacin de van der Waals (bsquela en algn libro de texto) relaciona las variables que
permiten definir el estado termodinmico de un gas real (Cules son esas variables?). En el
caso del CO2 la constante b es 4,3 10-5 m3/mol, y el valor de a que mejor ajusta la curva es a =
3,7 10-5 N m4/mol2.
a) Calcule la presin a 0C para el volumen especfico de 0,551 lt/mol, suponiendo que la
ecuacin es rigurosamente exacta.
b) Cul sera la presin, bajo estas mismas condiciones, si el CO 2 se comportara como gas
ideal?
Problema 14:
Usando la ecuacin de Van der Waals calcule el volumen que ocuparan 1.5 moles de (C 2H3)S a
105C y 0.750 atm. Suponga que a= 18.75 dm6atm mol-2 y b=0.1214 dm3mol-1

Bibliografa

F. W. Sears. Mecnica, Calor y Sonido. Captulo 23.


F. W. Sears. Mecnica, Calor y Movimiento ondulatorio. Captulo 21.
F. W. Sears.- M.W. Zemansky. Fsica. Cap. 18
N. Frank. Introduccin a Mecnica y Calor. Cap. 21
Young. Fundamentos de mecnica y calor.

Cu rso d e Fs ica I

Se gu nd o Cu a t rime st re de 20 14

50

FISICA I
MODULO II
CLASE 13
Objetivos

Analizar distintas mquinas trmicas y su rendimiento


Reconocer procesos reversibles.
Introducir el Segundo Principio de la Termodinmica

Problema 1:
Haciendo referencia al problema 7 de la clase 11 de este mdulo y:
a) Teniendo en cuenta el Primer Principio de la Termodinmica, demostrar que el rendimiento
del ciclo de Carnot es :
Q
1 c
Qh
b) Utilizando el resultado de a), demostrar que para un ciclo de Carnot se cumple la relacin:
Qc
T
c
Qh
Th

Para averiguar: Las relaciones anteriores dependen de la sustancia de trabajo de la mquina


trmica? Ver el ltimo problema de esta clase.
Problema 2:
Una mquina que utiliza un mol de gas ideal Cv = 5R/2 y Cp =7R/2, efecta un
ciclo que consta de 3 etapas.
Etapa 1: El gas se expande adiabticamente desde un volumen de 10 litros y una presin
inicial de 2,64 atm, hasta un volumen de 20 litros.
Etapa 2: El gas es comprimido a presin constante hasta su volumen original de 10 litros.
Etapa 3: El gas aumenta su temperatura a volumen constante hasta alcanzar su presin
original de 2,64 atm.
a) Haga el diagrama P V del ciclo.
b) Calcule el trabajo, la energa transferida en forma calor y la variacin de energa interna en
cada una de las etapas.
c) Determine si la energa se transfiere desde o hacia el sistema en cada caso
d) Calcule el rendimiento del ciclo.
Problema 3:
Una heladera sin freezer, para colocar bajo mesada, opera con un pequeo motor de
100 Watts de potencia. Si se la pudiera modelar como un refrigerador ideal de
Carnot, con temperaturas del foco fro y del foco caliente -5C y 20C
respectivamente:

Cu rso d e Fs ica I

Se gu nd o Cu a t rime st re de 20 14

51

a) Cunto hielo podra producir el refrigerador en una hora si se pusiera en su interior agua a
10 C?
b) Qu potencia debera tener el motor del refrigerador para producir la misma cantidad de
hielo en una hora, si la eficiencia real del refrigerador fuera el 50% del rendimiento de un
motor ideal funcionando entre las mismas temperaturas?
Problema 4:
Una mquina trmica trabaja sobre 3 moles de un gas ideal monoatmico, realizando el ciclo
reversible ABCD indicado en la figura. (a) Con los datos indicados, calcular las variables de
estado T, V y Eint en cada vrtice del ciclo. (b) Calcular el trabajo realizado sobre el gas en cada
etapa del ciclo. (c) Determinar el rendimiento de esta mquina trmica. (d) Describir cmo
podran llevarse a cabo experimentalmente cada uno de estos procesos (tener en cuenta que en
todos los casos se trata de procesos cuasiestticos).

Problema 5:
Calcule el rendimiento de un motor que funciona haciendo pasar gas
monoatmico por el siguiente ciclo:
a)
b)
a)
b)
c)

Comienza con n moles, a P0, V0 y T0.


A volumen V0 constante, aumenta su presin al doble.
A presin constante, duplica su volumen, alcanzando el estado P = 2 P 0, V = 2V0.
A volumen constante, disminuye su presin hasta llegar a la presin inicial P 0.
A presin constante, regresa al estado inicial P = P0, V = V0.

Considere Cv = 3 cal/mol K. Comience por hacer un diagrama del ciclo


Problema 6:
Un motor funciona con 0,1 moles de un gas ideal diatmico que realiza el siguiente ciclo:
comienza en un estado A en el cual la presin es 1 atm y la temperatura 300 K, sufre un proceso
a volumen constante hasta un estado B en el cual la temperatura es 600 K, luego una expansin
adiabtica hasta un estado C donde la presin es nuevamente de 1 atm, y finalmente una
compresin isobrica regresando al estado inicial A. (a) Hallar la presin y el volumen para los
estados A, B y C. (b) Calcular el trabajo neto realizado por el gas durante el ciclo. (c) Calcular
el rendimiento de esta mquina.

Cu rso d e Fs ica I

Se gu nd o Cu a t rime st re de 20 14

52

Problema 7:
Un motor extrae 250 J de un foco a 300 K y elimina 200 J en otro foco a 200 K. (a) Cul es su
rendimiento? (b) Qu cantidad de trabajo mecnico neto podra obtenerse en cada ciclo si se
tuviera un motor reversible que opere entre estos dos focos trmicos?
Problema 8:
Una mquina de Carnot funciona entre dos focos trmicos como refrigerador, tomando en cada
ciclo 100 J del foco fro y cediendo 150 J al foco caliente. (a) Determinar la eficiencia del
refrigerador, y el rendimiento de una mquina de Carnot que funcione entre estos mismos focos.
(b) Probar que un refrigerador que funcione entre estos focos no puede tener una eficiencia
mayor que la calculada en (a) sin violar la segunda ley de la termodinmica.
Problema 9:
Un gas ideal recorre un ciclo de Carnot formado por los procesos AB (adiabtico), BC
(isotrmico), CD (adiabtico) y DA (isotrmico). (a) Probar que los volmenes del gas en los
diferentes estados satisfacen la relacin VA/VB = VD/VC. (b) Mostrar que el rendimiento del
ciclo viene dado por = 1 T1/T2, donde T1 es la temperatura del foco fro y T2 la del foco
caliente.

Problema 10:
Una mquina trmica funciona con un gas real, y realiza el ciclo mostrado en la
figura, en la que se incluye la curva que indica las transiciones de fase. Es decir, consta de dos
procesos isotrmicos y dos adiabticos, como el ciclo de Carnot (Las versiones que estudiamos
anteriormente funcionaban con un gas ideal).
La eficiencia de esta mquina ser igual o menor al de
una mquina de Carnot que, funcionando entre las
mismas temperaturas, utilice un gas ideal?.

gas

lquido
vapor

gas

Bibliografa

Punte Gallardo Cotignola, II Principio de la Termodinmica. C.E.I.L.P.


Resnick Haliday (3ra edicin) Cap. 26.
Resnick Haliday Krane. Fisica vol I , 4ta Edicin Ed. Continental. 1993 Cap. 26.
N. Frank. Introduccin a Mecnica y Calor. Cap. 22
Young. Funda mentos de mecnica y calor.

Cu rso d e Fs ica I

Se gu nd o Cu a t rime st re de 20 14

53

FISICA I
MODULO II
CLASE 14
Objetivos:
- Revisin del Segundo Principio de la Termodinmica.
- Introduccin del concepto de Entropa.
Problema 1:
Un kilogramo de agua a 0C se calienta a 100C. Calcule su cambio de entropa.
Observacin: En la prctica el proceso descrito se efectuar de manera irreversible, quiz
colocando un recipiente con agua sobre una parrilla elctrica cuya superficie de coccin se
mantiene a 100C. Sin embargo, el cambio de entropa del agua slo depende de los estados
inicial y final del sistema, y es lo mismo si el proceso es reversible o si es irreversible.
Problema 2:
A un mol de gas ideal diatmico que se encuentra a 27C y una presin de 1 atm se le aumenta
su temperatura a presin constante hasta que triplica su volumen. Calcular (a) la variacin de su
energa interna, (b) el trabajo realizado sobre el gas, (c) la energa entregada al gas en forma de
calor y (d) la variacin de entropa del gas.
Problema 3:
Objetivo: Calcular el cambio de entropa en un proceso irreversible.
Para la maquina de Carnot, calcule el cambio de entropa total en la mquina durante un ciclo.
Qu magnitud tiene el cambio de entropa total del entorno de la mquina durante el ciclo?
Problema 4:
Se colocan 50 g de agua a 0 C en un congelador, cuyas paredes estn a una temperatura
aproximadamente constante de 10 C. Luego de un tiempo, el sistema alcanza el equilibrio
trmico. Calcular el cambio en la entropa del agua (luego hielo), mostrando que, si bien sta
disminuye, la entropa total del universo aumenta.
Problema 5:
Se encarga a tres ingenieros proyectar sendas mquinas trmicas. Las
indicaciones que reciben a tal fin son: la mquina debe absorber, de una fuente a
480C, 3340 kJ de energa en forma de calor por cada kg de fluido de trabajo,
siendo la fuente fra el ambiente a 17C.
Los proyectos presentados, ordenados segn la cantidad de trabajo que la mquina es capaz
de proporcionar por cada kg. de fluido de trabajo, fueron:

La mquina A, 2137,6 kJ
La mquina B, 2050,8 kJ

Cu rso d e Fs ica I

Se gu nd o Cu a t rime st re de 20 14

54

La mquina C, 1937,2 kJ

a) Cul de las mquinas fue aprobada? Explique su razonamiento


b) Verifique su respuesta a la parte a) mediante el clculo de la variacin de entropa, S.
Problema 6:
Dentro de un calormetro aislado se mezcla 1 kg de agua a 0C con 1 kg de
agua a 100C.
Halle la variacin de entropa del sistema Cules son las transformaciones reversibles
que debe idear para calcular esta variacin?
Problema 7:
Una pelota de 500 g, inicialmente en reposo, cae desde una altura de 2 m y rebota contra el
suelo, elevndose 1.5 m hasta quedar nuevamente en reposo. Calcular el cambio en la entropa
del universo si la temperatura ambiente es de 20 C.
Problema 8:
Una barra de acero a 500C, de capacidad calorfica 60 cal/K, se introduce en un bao de aceite
a 20C y con capacidad calorfica de 200 cal/K. Si el bao est aislado trmicamente, cul ser
el cambio de entropa en el universo?
Problema 9:
La figura representa un recipiente aislado, que consta de dos
partes (A y B), separadas por un tabique provisto de una vlvula C.
Los volmenes de A y B son iguales (1 litro cada uno). Inicialmente
la parte A contiene 0,2 moles de un gas ideal a una presin de 4 atm
y la parte B se encuentra en vaco. Cuando se abre la vlvula C el gas
pasa a ocupar todo el recipiente.
a) Hallar la temperatura final del sistema.
b) Calcular la variacin de entropa
ESTA ES LA LLAMADA EXPERIENCIA DE JOULE.
Problema 10:
Supongamos que se cuenta con una mquina trmica capaz de utilizar 1200
Joules de energa.
Dicha mquina se opera en dos condiciones distintas:
I) Los 1200 Joules son transferidos reversiblemente desde una fuente a 650 K.
II) Los 1200 Joules son transferidos a travs de una barra de cobre desde la fuente a 650 K a
otra a 350 K.
a) Determine el mximo trabajo mecnico obtenible de la mquina si en ambos caso la fuente
fra tiene una temperatura de 150 K.
b) Calcule la variacin de entropa total del universo en cada caso. Interprete en relacin con
la parte a).

Cu rso d e Fs ica I

Se gu nd o Cu a t rime st re de 20 14

55

Bibliografa

Punte Gallardo Cotignola, II Principio de la Termodinmica. C.E.I.L.P.


Resnick Haliday (3ra edicin). Cap. 25.
Resnick Haliday Krane. Fisica vol I , 4ta Edicin Ed. Continental. 1993 Cap. 26.
Tipler (2da edicin) Cap. 19.
Tipler , Fsica*, 3ra edicin, Ed. Revert, 1993. Cap 17
F. W. Sears. Mecnica, Calor y Sonido. Captulo 24.
F. W. Sears. Mecnica, Calor y Movimiento ondulatorio. Captulo 19.
F. W. Sears.- M.W. Zemansky. Fsica.
N. Frank. Introduccin a Mecnica y Calor. Cap. 22
Young. Fundamentos de mecnica y calor. Cap. 22

Cu rso d e Fs ica I

Se gu nd o Cu a t rime st re de 20 14

56

FSICA I
Prctica de laboratorio 3
Determinacin de la aceleracin de la gravedad en la ciudad
de La Plata
Objetivo
Determinar el valor de la aceleracin de la gravedad (g) y compararlo con el medido para la
ciudad de La Plata, por la Facultad de Ciencias Astronmicas y Geofsicas de la UNLP
Marco terico
Llamamos pndulo simple al sistema fsico constituido por una masa
modelada como partcula, sostenida desde un punto fijo por un hilo o varilla
de masa despreciable. Si se aparta el pndulo de su posicin de equilibrio,
el mismo oscilar repitiendo el movimiento en forma peridica. Se llama
perodo (T) de dicho movimiento al tiempo requerido para realizar una
oscilacin completa (pasar consecutivamente por la misma posicin
movindose en la misma direccin). Repasemos brevemente el modelo.
Vamos a considerar entonces que toda la masa del pndulo se encuentra
concentrada en el centro de la esfera, y que la cuerda de la que pende la
esfera es flexible pero inextensible, y de masa despreciable. Las fuerzas que
actan sobre la esfera son entonces su peso y la tensin de la cuerda.
Supondremos adems que el movimiento est restringido a un plano y
vamos a despreciar el efecto del rozamiento de las partes mviles con el
aire.
Las ecuaciones de movimiento en la direccin tangencial al arco

Figura 1

d 2
ml 2 mgsen
dt
o
si el ngulo es pequeo, 5 , se puede aproximar el seno por su argumento(en radianes).
En esa aproximacin, la solucin de la ecuacin diferencial ser, tomando como condicin de
inicial para t=0 partimos del reposo con la masa en x=x0,

(t ) 0 cos

g
t
l

por lo que la aproximacin es consistente para todo t. Esta es una solucin peridica de perodo

T 2

L
g

Cu rso d e Fs ica I

Se gu nd o Cu a t rime st re de 20 14

57

Materia les

Hilo de nylon
Soporte
Esfera de plomo
Photogate Vernier
Cinta mtrica

Procedimiento

Determinar la longitud del pndulo (hilo + CM de la esfera), realizando una medida.


Tomar entre 0,8 m y 1 m.
Apartar el pndulo de su posicin de equilibrio de manera que el hilo se encuentre a
aproximadamente 5 grados de la vertical, por ejemplo para un hilo de 50 cm de largo,
apartar la esfera 5 cm hacia un lado respecto de la posicin de equilibrio, si utiliza otra
longitud realizar el clculo.
Tomar el periodo utilizando el sensor en modo pndulo. Este procedimiento se repetir
5 veces y en cada medida se relevarn 20 oscilaciones del pndulo.

Resu ltados
Esfera plo mo
Medida
1
2
3
4
5
T T

E sfera de plo mo

Perod o

Medida
1
2
3
4
5
T T

Perodo

Discu sin y conclusiones

Utilizando el modelo terico determinar el valor de g y su incerteza a partir de los


periodos medidos en cada experiencia.
Calcular la exactitud de cada medida utilizando como referencia el valor medido en el
Observatorio: g= (9,7973667 0.0000001)m/s2

Donde M1 es el valor calculado en el laboratorio y Mref es el valor de referencia

Cu rso d e Fs ica I

Se gu nd o Cu a t rime st re de 20 14

58

Biblio gra fa para con sultar:


In troducci n a las Prcticas de Labora torio de Fsica I
Clase 0. Incertezas y Medidas.Fisica I.PPT

Cu rso d e Fs ica I

Se gu nd o Cu a t rime st re de 20 14

59

FSICA I
Prctica de laboratorio 4
Motivacin
En algunos procedimientos mdicos es necesario determinar la densidad de un paciente para
saber la relacin entre masa muscular y materia grasa. En estos casos, el desafo es determinar
el volumen de lquido desplazado por el paciente ya que se lo sumerge una piscina, suspendido
en un arns colgado de un dinammetro. Veremos a continuacin dos procedimientos en donde
se determina la densidad, en uno de ellos sin medir el volumen de agua desplazada.

Objetivo
Determinacin de la densidad de un cilindro metlico empleando un procedimiento con dos
variantes para su clculo:
a) Con una medida directa del volumen desplazado.
b) Con una medida indirecta del volumen desplazado.

Materiales e instrumentos

Dinammetro o sensor de fuerza (registrar rango y resolucin)


Vaso graduado (registrar rango y resolucin)
Cilindro de metal
Agua
Calibre
Balanza
Sensor de fuerza

Modelo Terico
Principio de Arqumedes
El principio de Arqumedes es un principio fsico que afirma que: Un cuerpo total o
parcialmente sumergido en un fluido en reposo, recibe un empuje de abajo hacia arriba igual
al peso del volumen del fluido que desaloja. Esta fuerza recibe el nombre de empuje
hidrosttico o de Arqumedes, y se mide en newtons (en el SIU). El principio de Arqumedes se
formula as:
E= m.g =
Donde E es el empuje,
es la densidad del fluido, V el volumen de fluido desplazado por
algn cuerpo sumergido parcial o totalmente en el mismo, g la aceleracin de la
gravedad y m la masa, de este modo, el empuje depende de la densidad del fluido, del volumen
del cuerpo y de la gravedad existente en ese lugar. El empuje (en condiciones normales y
descritas de modo simplificado) acta verticalmente hacia arriba y est aplicado en el centro de
gravedad del fluido desalojado por el cuerpo.

Cu rso d e Fs ica I

Se gu nd o Cu a t rime st re de 20 14

60

Ejemplo del Principio de


Arqumedes:
El
volumen
adicional en la segunda probeta
corresponde
al
volumen desplazado
por
el
slido
sumergido
(que
naturalmente coincide con el
volumen del slido).

Diagramas de cuerpo libre de las distintas configuraciones

Procedimiento
El primer paso consta de colgar el cilindro al sensor de fuerza y anotar el dato que dar la
interfaz desde la pc. A ese valor se lo denominar
(Corresponde a la indicacin del
instrumento con el cuerpo en el aire)

tendr una incerteza asociada que se calcular de manera directa con el programa
mediante un anlisis estadstico de la funcin fuerza vs tiempo.
Por lo tanto obtendremos la siguiente informacin:

Cu rso d e Fs ica I

Se gu nd o Cu a t rime st re de 20 14

61

El segundo paso consistir en sumergir el cilindro en un vaso graduado


que contenga agua sin soltarlo del sensor de fuerza y evitando que toque
el fondo del recipiente. Se registrar el desplazamiento del volumen de
agua obtenindose as el volumen del cilindro (
Tambin se deber
anotar el dato que indique la interfaz para esta situacin al cual se
llamar
(Corresponde a la indicacin del instrumento con el
cuerpo en el agua).
De la misma forma que en el primer paso con ayuda del programa
se anotara:
Respecto a la incerteza en la medida del volumen la misma estar dada por la graduacin del
vaso:
Nota: El procedimiento anterior tambin se llevar a cabo utilizando un dinammetro en lugar
del sensor. En este caso la operacin uno se llevar a cabo 5 veces y la operacin dos al menos 3
veces. Para este caso:

Donde

Una vez obtenidas todas las mediciones se proceder a calcular la densidad del cilindro
empleando dos variantes.
Nota: El empleo del dinammetro estar sujeto a la disponibilidad del material durante la
prctica del laboratorio
Variante 1
Conociendo el volumen desplazado
y el peso del cuerpo sumergido en aire (
)
Modelo terico 1
La densidad del cilindro de metal
empleando la medida del sensor o del dinammetro
(Tcaire) y del volumen medido con el vaso graduado (Vc) es:
(1)
Su incertidumbre, al ser una medida indirecta, se hallar por el mtodo de propagacin de
errores:
(2)
Variante 2
Conociendo las medidas del instrumento con el cuerpo sumergido en aire (

) y en agua (

) respectivamente.
Modelo terico 2
Aplicando la 2 ley de Newton

Cu rso d e Fs ica I

Se gu nd o Cu a t rime st re de 20 14

62

siendo E=empuje. Sabiendo que E=

podemos estimar el

volumen del cuerpo:

Luego se calcula su incertidumbre por propagacin:

Por consiguiente la densidad del cilindro empleando las medidas de

es:

(3)
Y su correspondiente incertidumbre (considerar la densidad del agua con incertidumbre nula):

(4)
Nota: Utilizar el valor de g en la ciudad de La Plata: 9.79

1. Comparar ambos resultados contra valores tabulados y citar la bibliografa. Calcular la


exactitud de la medida con la siguiente frmula:

Donde

= medida experimental y

= medida de referencia (valor tabulado)

Anlisis y discusin de resultados


Por qu ignoramos el empuje del aire al tomar nuestro primer dato con el objeto
colgado en el sensor de fuerza en el aire?
Importa la profundidad del slido en el agua?
Qu variante conduce a un resultado ms exacto? Es aceptable resignar exactitud?

Cu rso d e Fs ica I

Se gu nd o Cu a t rime st re de 20 14

63

Vous aimerez peut-être aussi